Está en la página 1de 99

27 6 FffiJi:i'J,,'orFruid Frow

FLUID
&
iLUID MECHANTCS
Fluid Flow
CHAPTER SIX
^,E
Measurement Z I I
3 HYDRAULICS
Problem 5 - 37
High velocity water flows.up-an-inclined plane, as
What are the two possible depth of flow at section
showp in Figure
2? Negrectar loies.
Ans:0.775 m &2.74
5,
Chapter 6
Fluid Flow Measurement
I

i There are numerous number of devices used to measure the flow of fluids. In
Figure 5 - 11 lny of these devices, the Bernoulli's Energy Theorem is greatly utilized and
ldditional knowledge of the characteristics and coefficients of each device is
lnrportant. In the absence of reliable values and coefficients, a device should
9.806 m/s hc calibrated for the expected operating conditions.

DEVICE COEFFICIENTS

Coefficient of Discharge, C or C6
'['he coefficient of discharge is the ratio of the aptual discharge t]r,rough the
clevice to the ideal or theoretical discharge which would occur without losses.
'[his may be expressed as:

CorCa:; Actualdischaree
------------'--o- O
' Theoreticpldischarge=+ Qr
Eq.6-1
-a'-

The actual discharge may be accomplished by series of observation, usually by


measuring the total amount of fluid passing through the device for a known
period, The theoretical value can be accomplished using the Bernoulli,s
Theorem neglecting losses.

Coefficient of Vetocity, C,
The coefficient of velocity is the ratio of the actual mean velocity to the ideal or
theoretical velocity which would occur without any losses.

Actualvelocily U
Eq,6-Z
27 g F,Xi'J,:l'l[asurement ILUID MECHANICS CHAPTETT SIX
27q
& HYDRAULICS Fluid Flow Measurement
Coefficient of Contraction, C"
Table 6 - l: Discharge Coefficients for Vertical Sharp-Edged Circular Orifice
The. coefficient of conkaction is the ratio of the actual area of the conh Discharging into Air at 15.6"C (60.F)
section of the stream or jet to the area of the opening
through which the
flows. Head Diameter in mm
(m) 6.25 12.50 18.75 25.00 50.00 100
0.24 0.@7 0.627 0.515 0.609 0.603 0.601
0.43 0.535 0.619 0.610 0.605 0.601 0.600
0.61 0.529 0.615 0.507 0.603 0.600 0.599
1.22 0.621 0.609 0.603 0.600 0.s98 0.597
1.83 0.6L7 0.607 0.601 0.599 0.597 0.s95
2.44 0.614 0.60s 0.600 0.s98 0.596 0.595
Retationship between the Three Coefficients
3.05 0.613 0.604 0.600 0.597 0.596 0.595
3.66 0.612 0.603 0.599 0.597 0.595 0.595
Actualdischarge, Q= C"e, ) Eq (1)
4.27
4.88
0,611 o 6n? 0.598 0.596 0.595 0.594
0.610 0.602 0.598 0.596 0.595 0.594
Also 5.10 0.609 0.602 0.598 0.596 0.595 0.594
Q = Actual area, a x Actual velocity, zr
7.62 0.608 0.501 0.597 0.s96 0.594 0.594
Q=C,AxC,,a1 9.15 0.607 0.600 0.597 0.595 0.594 0.594
t2.20 0.606 0.500 0.595 0.595 0.594 0.593
Q= C,C,,Aa, 15.24 0.605 0.599 0.s96 0.595 0.594 0.593
but Aas= Q, 18.30 0.505 0.599 0.596 0.594 0.593 0.593
Q=C,C,QI )Eq.(2)
From Equations (1) and (2)

HEAD LOST
'l'he head lost through Venturi meters, orifices, tubes, and nozzles may be
The coefficient of discharge varies with Reynords expressed as:
Number. It is not cons
for a given device. Tabre 6 - 1 gives the coefficients for
vertical rt urp
orifice. "i
Figure 6 - 2

The ideal energy equation between 1 and 2 is:


Er=Ez
Q * 0.62
C; ! 0.98 c" t.Pl +rr=4alLar,
29v29v
Cd - 0,51 c,
Ct I
Arar = Azaz
Square shoulder Thick plate
A2 atz
a,=
' A1 -
crr and =(!a\'or'
Figure 6 - 1: Orifice coefficients 29 [ArJ 29
28O F,Xfi?:l',nasurement FLUID MECHANICS
I HYDRAULICS Fruid Frow,I:*il.'I:ff 281

ef *.?*o=**?*,, HL=
o2

+ *1, (f)'l
2g
[,[*)']
=(?.,,).(+.,,)
f, t+)'] * u,=l#-,i[,-(*)')* E..6.5
(zr2)th@reticar
) (a)
@ If the orifice or nozzle takes off directly from a tank where .{1 is very much
greater than Az then the velocity of approach is negligible and Eq. 5 - 5
Considering head lost between 1 and 2: reduces to:

{.zgY2gy +21-HL= uz, + p2 *,^


P7

nr-(r;--r)* Eq6 -6
This equation sirnplifies to:

(t'r)".,,ur Note: a=acfualvelocity


) (D)

Since-4u.6"1 =

a=c'f,-#FW
Cp ?theoreticat
=? ORIFICE
An oifice is an opening (usually circular) with a closed perimeter through
which fluid flows. It is used primarily to measure or to conkol the flow of
fluid. The upstream face of the orifice may be rounded or sharp. An orifice
with prolonged side, such as a piece of pipe, having a length of two or three
Squaring both sides_and arranging terms:
times its diameter, is called a short tube. Longer tubes.such as culverts under
embankments are usually treated as orifice although they may also be treated
* l, (f)'] + (+-.,,). (?.,,)
= as slwrt pipes.

From (D) According to shape, orifice may be circular, square, or rectanlular in cross-
section. The circular sharp-crested orifice is most widely used because of &e
(+.,,) (?.,,)= * (f)'] .". simplicity of its design and construction.

f, The figure telow shows a general case of fluid flow through an orifice. Let pa
andps be the air pressures in the chambers A and B, respectively and aa be the
* l, (*l'l # * =
l,
-(*)'1. ", velocity of the stream normal to the plane of the orifice (velocity of approach).
Consider two points 1 and 2 such that ot = ua and o2 = r, and writing the
energy equation between these two points neglecting losses:
,cD-
loz
CHAPTER Stx
Fluid Flow Measurement FLUID FIUID MECHANICS CHAPTER SIX -t.r rr
& & HYDRAULICS Fluid Flow Measurement ZOJ

Where H is the totalheadproducing fluo in meters or feet of the flowingfluid. It


can be noted in Eq. (1) that H is the sum of the flow energy upstream less the
flow energy downstream, or

H,= Head Unstream - Flead DoWnstream Eq.6 -12

Values of H for Various Conditions

Chamber A
Chamber B
a
Energy equation between 1 and 2 neglecting head A
lost
Et=Ez I
I
uz, * lZ *,"
t29 - Pt
y +z.t= zs'y "'
I

. pa+vtt *o= !
+29y2gy" * pB
*o
p^ a2
{.zgY2gy*lr_ * ps

,=w
t=t*P^
2gyy29 -Ps *'ui

H=h+p/y H=hz+htkrly)+plyz
zB4 F,Xf,'J:I'^nasurement MECHANICS
F,uidFrow;l*il"T:I 285
under Low Heads
the head on a vertical orifice is small in comparison with the height of
orlfice, there is an appreciable differencebetween the discharges using the
analysis.

the rectangular section of length L and height D as shown in the


Submerged Oriflce
(Neglecting v.)
with both the surface and the jet subject to atmospheric pressure. The
discharge through an elementary strip of length L and height d/r is:
H=hr-h:=h

lQt= (L dh) Jzsh


Contraction of the Jet dQ, = Jzg Llt/z dlt
lrz
of nuid now through a verrical
H:,j,_::X1,#:-."f:.:"::,
sharp-edged l1o.ss.se1.tion
orifice frorn a reservoir utiffi;:Hffir?#:lTil Um
to *,u -G Q,= JZS, t
::,H:1*::"
orifice,
j,::^.,y:1.
:'l make an1q"
they cannot
o"1"1 r,o.,. ir,"
ibruptchange;
o.in* il -

;,
ilr'.;;;;:""Jl?U";:;"rl:the
reave
curvilinear paths, thus causi.g trie jet
toiontract
j::#.::
for a
short aistarr.e beyoncr
the corttraction of ttrc jet rhe
e,= lzs rllrr)',',
,#Ti::::.1,.:
section on the jet where rhe l: Ti"*ua'io'u,
conrracrior,
Lr'"TtJi"rr)if, ,
located ar one ""ur"r-lr;ffiffi
r,rir.iin".,,rn" *,*Zr!iirtrl ,,
Q,= ? Jzg t thf lz - 7rt1z 1

il:tH:Iiroximately from
Q=CQ,

a= +cJzsLfur'/'-hr'/' Eq.6-13

VENTURI METER
Veturi meter is an instrument used in measuring the discharge through pipes.
It consist of a converging tube AB (see F'igure 6 - 3) which is connected to the
main pipe at the inlet at A, and ending in a cylindrical section BC called the
tfuoat, and a diverging section CD which is connected again to the main pipe
at the ouflet D. The angle of divergence is kept small to reduce the headlost
cause by turbulence as the velocity is reduced
286 F,:trJ,:I'ffiasurement FLUID M ILUID MECHANICS
& A HYDRAULICS Fruid F.w,fi:*f:*:H 287
,rT- The theoretical or ideal discharge " Q, " cart be found onie oi or xr2 is known.
Tlre actual discharge "Q" is computed by multiplying the theoretical value by
I

the coefficient of discharge or meter coefficient " C' .


T-_ --l-
rlth
I

Q=CrQt Eq.6-1.4
I
L
Prlv
+\ Note: If we neglect the head lost in our energy equation, the values we get are known as
theoretical or ideal values (theoreflcal velocity and theoretical discharge). Considering head
lost, we getthe adual ualues(afr,nl velocity and actual discharge).

NOZZLE
A nozAe is a converging tube installed at the end of a pipe or hose for the
purpose of increasing the velocity of the issuing jet.

Zr Piezometer ring 22
0z<0r
I

-.-._v_____.__.___
Figure 6 - S: Venturi meter

Consider two points in the system, 0 at


the base of the inlet and O at the
and writing the energy equation between these
.fu.1:, two points neglecting The discharge through anozzle can be calculated using the equation
head lost:
,r' * p, u^2
L:-
P.>
+
29v ,28 v
"^ Q=CA,,,[TgH Eq.5-15
,22 ot2
2g - 2g
= (+.,,) -(u*,,)
\v ) where:
H = total head at base of nozzle
A, = area at the nozzle tip
si$e- of the equation is.the.kinetic energy which shows an increase
T"-tt
while the left side of in varue,
the equation is the potenfal energy *t ict
Therefore
,io*l de"r"u" i,
lalue' head rost, the inc:rease in.kinetic utergy is The following table gives the mean values of coefficients for water discharging
^egrecting equal to ilu
decrease in potentiar mergy. This statement
is known as rhe venfui principre. through anozzf,e having a base diameter of 40 mm and C. = 1.0.
The difference in pressure between the inlet
and the throat is commonry measured
by means of a differential ma.ometer connecting
th"
-
;;;i,hr;;;. Tip Diameter in mm 19 22 25 29 32 35
lf the elevations and the difference in pressure
between o and o are knowrL the C=C, 0.983 0.982 0.980 0.976 0.971 0.959
discharge can be solved

The head lost through anozzle is given by Eq. 5 - 5.


2 88 F,Xfi'J,:['^flasurement FLUID M
&
FLUID MECHANICS
& HYDRAULICS Fruid Frow;:*:LT:tr 289
PITOT TUBE
This equation shows that the velocity head at point 1 is transformed into
Named after the French physicist and
e,gineer Henri pitot, pitot tube is a I pressure head at point 2.
(L-shaped"o_r U-shaped) tube with
b.th and is used to measure
vetocity of fluid flow or verocity orui, ";;; "pen
n* ur;ffi ;;;;;"-r;""$:;:[:
(a) (b)
when the tube is praced in.a moving stream
with open end oriented into tl
now, the Iiquid tf,.ofuJ.g
lT":Y rises a distance of""t"r,
the tube 1r rr above ,t" ,t
ut point 2 untit the ,rrfr"" I
surface. o"
condition is the' establisr-recr, and the
q"ir-,lty"uro
unchanged as the fl ow remai.s steady.' poini,
"d;ff;#i
of liquid in the tube remains
in:i"*'
"i ";'il"t1;iriffii I
ph

Figure 6 - 5

Figure 6 - 4

Consider a particle at point 1 to. moving


with a velocity of a. As the particle
approaches point 2, t=v10c1tr is.gradriary
retarded to 0 at point 2. writing
the energy equation between 1-and"2
di;Jfi"g friction:
il=Ez
L *r/= 92. P, *,/
{.zgy/29yr
Pt b^
al=0) = ht; _ =h2
v v
,r2
:- +h=lh
zg
u2=2g(fu-h2'1
Figure 6 - 6: Pitot tube in a pipe
29 O F,:f,'J,:l'ffiasurement
FLUID MECHANICS
& HYDIIAULICS;
FLUID MECHANICS
& HYDRAULICS Fluid Flow
CHAPTER StX )a I
Measurement L ' '
GATES
A gate is an opening in a dam or other hydraulic structure to control Actual o = C,J zs(d, - d2)+ arz Eq,6-18
passage of water. It has the same hydraulic properties as the orifice. In us
gates, calibration test are advisable if accurate measurements are to be Actual Q = CA J ZS@, - d2) + o12 F,q.6.\9
obtained since its coefficient of discharge varies widely.
Coefficient of conhactio n, Cr= Eq.6 -20
The following illustrations show the two different flow conditions through the
!
sluice gate.
where:
C = C, G (varies from 0.61 to 0.91)
a=by
b= width of the flume

TUBES

Standard Shoft Tube


A standard short tube is the one with a square-cornered entrance and has a
Figure 6 - 7 (a): Free Flow tength of about 2.5 times its internal diameter as shown in Figure 5 - 8. Figure
Figure 6 - 7 (b):Submerged Flow
6 - 8 (a) shows a condition when the flow started suddenly with high heads so
Figure 6 - 7: Flow through a gate that the jet may not touch the walls of the pipe. This condition is very much
the same as that of a sharp-crested orifice. Figure 6 - 8 (b) shows a condition
In Figure 6 - 7 (a), writing the e.ergy equation between 1 and 2 negrecting when the jet touches the walls of the tube. The discharge through this tube is
head lost: about one-third greater than that of the standard sharp-edged orifice but the
il= Ez velocity of flow is lesser.
,1

!.P7 +zt=r22 +P2 +2,


2gy2gy
where lL = dr and Pz Q=1.0
v v C = C,= 0.82

o,2 ,r^2
*a,*o
;i.dt+o=i;
')r.
7)
^- 7)
"'
=h-dz
,t ,r
?t22-Pf==2g@t-d.z)
az2 = 28 (d1- *) + pf
Figure6-8(a) Figure6-8(b)

Figure 6 - 8: Standard Shott tubes


29 2 F,Xf,J,:Irffiasurement FLUID MECHANICS
& HYDRAULICS.T
FLUID MECHANICS
& HYDRAULICS FruidFrow;:flI"T:f 293
Converging Tubes
Re-entrant Tubes
Conical converging tubes has the form of a
Tlrese are tube having their ends projecting inside a reservoir or tank
frustum of a right circular cone with the
larger end adjacent to the tank or reservoir
as shown in Figure 6 - 9.

Borda's Moathpiece - This is a special case of a re-entrant tube,


consisting of a thin tube projecting into a tank having a length of
about one diameter. The coefficient of contraction for this tube,
C. = 0.5 and C, = 1.0.

Submerged Tubes
Table 6 - 2: Coefficients for Conical Converging Tubes An example of submerged tube is a culvert conveying water through
embankments. The discharge through a submerged tube is given by the
formula:

Q=CAW Eq,6-23

Where C is the coefficient of discharge, A is the area of the opening, and H is


the difference in elevation of the liquid surfaces.
Diverging Tubes
A diverging tube has the form of a frustum of a righf circular cone
with the
smaller end adjacent to the reservoir or tank.

Figure 6 - 1O: Submerged Tube (Culvert)


29 4 Fllf,'J,:l'ffiasurement FLUID MECHANICS
& HYDRAULICS F'uidF,owfi:*il:I:fi 295
UNSTfADY FLOW
Asdh
The flow through orifice, weirs, or tubes is said to be steady onry if the t, dt=
head producing flow, H, is constant. The amount of fluid being discharged Qin-Qorr,
a time f can therefore be computed using the formula

,=l:,'# Eq.6-25

where Q is the discharge, which is constant or steady. In some conditi


howeyer, the head over an orifice, tube or weir may .,ruiy u, the fluid flows When there is no inflow (Qi, = 0), the formula becomes:
and thus causing the flow to be unsteady A, dl,
'- Jr,fh' -a"-
lnterchanging the limits to change the sign of the integrand:

,= Eq.6 - 26
r;+#
Note: If A, is variable, it must be expressed in terms of h.

Figure 6 - l1
If the outflow is through and orifices or tube, = CA
Qout If the flow is
"lrp".
through any other openings, use the corresponding formula for discharge.

For tanks with constant cross-


sectional area and the outflow is
Consider the tank shown in the figure to be supplied with a fluid (inflow) through an orifice or tube (with
a
sin'rultaneously discharging through an outlet (either an orifice, t,be, weir no inflow), the time for the head
pipe). Obviously, if Q,n > po,u the head will rise and if
eo,1 > e,n, the head wid to change from Hr to Hz is:
fall' suppose we are required to compute the time to rower the revel from /lr tol
/zz (assuming'Qo,t >
Q1n), the amount or Ruia which is Iost in the tank will be
ardH
dv
't= SH'CA,r[TgH
= (Q,"- Q.,r) df Jr,
at= dV
Qin-Qo,, ,=# [:j,,,,,,
where dv is the differential volume lost over a differential time df.
,
If the head
over the outlet is /2, then the level will drop ilh, thus dv = A, dh whereA,
surface area in the reservoir at any instant and may be constant or variable,
is the ,=#[r"*],,
then
296 CHAPTER SIX
FLUID MECHANICS
Fluid Flow Measurement
& HYDRAULICS
l FLUID MECHANICS
& HYDRAULICS Fruid F.w,;:*il:I,:fi 297
WEIR
Weirs are overflow structures which are built across an open channel for the
purpose of measuring or controlling the flow of liquids. weirs have been
commonly used to measure the flow of water, but it is now being adopted to
If liquid flows throueh a measure the flow of other liquids. The formulas and principles that will be
s,,h-o--^r ^-:r:^^ ^- L.L^ -^---^-d---.-.--
tliscussed on this chapter are general, i.e. applicable to any type of liquid.
:.n"il,*"n*",",i,tJfl
these formulas
:;if :ffi:i#'Jtr;l,T?r:;:tJS:fl:,Jiln:;
36t "-
pROBLEM
in _
6

Classification of Weirs
According to shape, weirs may. be rectangular, tinngular, trapezoidal, circular,
parabolic, or of any other regular form. The most commonly used shaPes are
the rectangular, triangular and the hapezoidal qhapes. According to the form
of the crest, weirs, may be slurp-crested ot broad-crested.

'[he flow over a weir may either be free or sabmerged. If the water surface
downstream from the weir is lower than the crest the flow is free, but if the
downsheam surface is higher than the crest, the flow is submerged.

Definition of Terms
Nappe - the overflowing stream in a weir.
Crest of weir - the edge or top surface of a weir with which the flowing
Iiquid comes in contact.
Contracted ueir -
werrs having sides sharp-edged, so that the nappe is
contracted in width qr having end contractions, either one end or
two ends.
suppressedrueir or full-toidthzoeir - weirs having its length L being equal to
where Ast'and A,z is the
water surface areas in the tanks at any time, and H is , the width of the channel so that the nappe suffers no end
the difference in water
surfaces in the two.tanks aL urrE' ,If ,'rsl conkactions.
will vary, it must be expressed
in terms of H.
^ at ary
any time. A,r dand,/ or A"z
Drop-dorun culoe - the downward curvature of the liquid surface before the
If A'r and A52 dr€ constant, weir.
i.e. the two tanks have uniform cross-sectionar area,
the formula becomes: Head, H - the distance between the liquid surface and the crest of the weir,
, measured before the drop-down curve.
29 I F,XtrJ,:['ilasurement
FLUID MECHANICS
& HYDRAULICST
FLUID MECHANICS
& HYDRAULTCS Fluid Flow
CHAPTER SrX
?oo
Measurement L , ,

Ioa,=
1O, Ir
.troftdtt

.,2,
e,= Jzs rlif.r,)*];
e,= tr JrS tt(H + h,)3/2 - (0 + txo)t/21

Actual Q=CQ,

Q= Zc,E r l(a+ h,)'/'-h,3/z Eq.6-30

It is a common practice to combine Z Jrg C into a single coefficient C,, called


the ueir factor. The general formula for a discharge through a rectangular weir
considering velocity of approach then becomes

RECTANGULAR WEIR Q = c,,L l(H +n,)z/z -h,3/z Eq.6-31

If the ratio H/P is sufficiently small, the velocity of approach becomes very
small and the term hrf /2 rnay be neglected. The discharge formula become$

Q= CnuLHtr Eq.6-32

In situations where the discharge is required considering the velocity of


approach, using Eq. 6 - 30 or Eq. 6 - 31 would lead to successive trials to solve
Figure 6 - 13: Section A_A of Figure 6 - 12. for Q (since the velocity of approach ho is a function of Q). The following
simplified equation may be used:
Consider a differential area of length
L and heigh t dlt to be located h meters
ffi,;;;"};r,,rquid
surface. By orifi?e il; thJtheoretic"i ;;i;;il'rh.d;
Eq.6-33
u,= JTfr
where the total head oroducing
flow H' = rr + rto,where ft, is the verocity head Eq.6-34
of approach and is equal to i,r/Zg. The
uuvuE'rr ,n"
discharge through urs t
skiP is then, "i"rJ""t"ri i

dQt= dAot
where d = depth of water upstream
dQ,=LdhJrg\t+U) d=H+P
3OO F,:trJ:l'ffiasurehent FLUID MECHANICS
T HYDRAULICS Fruid Frow,fi:*il5fffi 30 t
Standard Weir
BAZIN FORMULA
The following specifications must be applied to a standard
rectangular For rectangular weirs of length from 0.5 m to 2.0 m under heads from 50
without end conhactions:
mm to 600 mm.
1' The upstream face of the weir plate should be verticar and smooth.
2' The crest edge shall be lever, shan have a square upstrean"r corner,
and shall be narrow that the water will ,,ot tor.h it again c,=ossie(a.rnr*
o'*") Eq 6-40
after
passing the upstrearn corner. ['.r"(#)']
The sides of the flume shall be vertical and smooth and
shal extend
a short distance downstream past the weir crest.
The pressure under the nappe shall be atmospheric.
Contracted Recta n gular Wei rs
5. The approach cha.nel shalr be of uniform cross section
for a sufficient The effective length of L of a contracted weir is given by:
distance above the weir, or shar be provided with
baffles-,h;a;
normal distributio. of velocities exists in the flow upprou.rrirrg?l
weir, and the water surface is free of waves or surges... L=L'- 0.1NH Eq,5-41

Standard Weir Factor (C*) Formulas where L'= measure length of crest
N = number of end contraction (1 or 2)
Numerous equaLions
coefficient Jyvl bgen developed for finding the discharge
C,, to ue used i. Eq. 6.- 31 and Eq o 32. some
H = measured head
, Jf th"ru are given
below.

FRANCIS FORMULA
upon experiments on rectangular weirs frorn1.07 m (3.5
!-1ted ft.) to 5.1g m
(17 tf.) long under heads from 1g0 mm to 490
mm.

For H/ P < 0.4, the following value of C,,, may be used. One-end Contraction Two-end Contraction
(N=1) (N=2)

TRTANGULAR WErR (V-NOTCH)


REHBOCK AND CHOW FORMULA At very low heads, the nappe of a rectangular weir has a tendency to adhere to
its downskeam face. A weir operating under such condition would give a
very inaccurate result. For very low heads, a V-notch weir should be used is
accuracy of measurement is required. The vertex angle 0 of a V-notch weir is
usually between 10q to 90o but rarely larger.
3O2 F,X*'J,:ffffiasurement FLUID MECHANICS
,uia rrowrfiI*il:f:H 3o3
& HYDRAUI-ICS

r_ a2 o= * J8 {"E"tn)-zssrzl-ol
a- * lrs (#r','\
e = * ,[S tWr, (theoretical e)

Actual Q- Cx Qt

p-.frc.!-zgtwz Eq.6-42

Eq.6 - 42 can be used even if the side inclinations are unequal.

figure 6 - 14; Triangular (V-Notch) weir For triangular V-notch weir,

, 'lt The.discharge through the differential strip is:


dQ=odA
tan(o/2)=
+
I
L =2H tnr (0/2)
= pfn (neglecting velocity of approach)
" then,
dA=xdh
Q= # CJzg l2H tan(o/2)llttrz
, by similar kiangles:
x _L
. HJ_ H
AA
t
Q= + c,,!zg tanll'1s/z= (,otaniusn Eq.6 - 43
x- " (H-li
H'
I
dA= a(H_lt)dlt
For standard 90o weir:
de= ,,l2gh
-t fitu_n)an
t- ,.,' . ',, a=l.{lts/z(S.I.Units) Eq.6-44
de.= ,f z6 n,r, (H _ h) dh
; , . :' Q-2.5115/2'(English) Eq.6-45
I-
do = - h3/2) dh
;'tzg 1utl'r'
Integrate

,= * J4 [@',' -rrrzhh
0

a=* [r(+r',')-ir,u/,t
30 4 F,Xfi'J:l',nasuremenr FLUID MECHANICS FLUID MECHANICS
& HYDRAULICS & HYDRAUTICS F,uid Frow,fi:flI5fl:H 305
TRAPEZOIDAL SHARP CRESTED WEIR
The discharge frorn a trapezoidar weir is SUTTRO WEIR OR PROPORTIONAL FLOW WEIR
assumed the same as that from a It may be noted that, in a rectangular weir, discharge varies with 3/2power of
rectangular weir and a triangular weir
in corabination
H and, in a triangular weir, with5/2 power of H. There exist a shape for
which'the discharge varies linearly with the head, the proportional floru or
Rettger weir, also known as Suttro weir.
!
t-

Hyperbolic
L

Figure 6 - 15: Trapezoidal sharp-crested


weir

where Z= +,subsrituted for tan


--"' $ i.,
Q=1/zCnK,!2gH Eq.b-49
H' 2 "' Eq. O _ +e.
X= zx,tV Eq. 6 - 50

CIPOLLETTI WEIR SUBMERGED SHARP WEIR


Cipolletti weirs are kapezoidar weirs with The discharge over a submerged sharp-crested weir is affected not only by the
side srope of t horizontar to 4
vertical' The additionSl area at the sides head on the upstream side Hr but by the head downstream Hz. The discharge
adds approximatery enough effective
width of the stream to offset the side contractions. for a submerged weir is related to the free or unsubmerged disCharge.
Villemonte expressed this relationship by the equation

0 = 75.96,
a ='14.04,
3=[,(ff)']':' Eq.6-51

Submerged sharp-crested weir


.II,^ CHAPTER SIX
5UO' Ftuid Flow Measurement
FLUID MECHANICS
& HYDRAULICS
FLUID MECHANICS
& HYDRAULICS Fruid F.w,;:fl[[:H 3ol
where n is the exponent of H in the equation for free discharge for the shape of
weir rrsed. For rectangular weir, n:3/2 and n = 5/2for triangular weir. Solved Problems
Problem 6 - 1
UNSTEADY FLOW WEIR (VARIABLE HEAD) A volumekic tank 1.20 m in'diameter and 1.50 m high was filled with oil in 16
minutes and 32.4 seconds. \A/hat is the average discharge?
or tank
constant water
surface area, A.
Solution

/.: Discharge, 0 = Volume


n*"
:

Discharge, a= iG.2)2O.5)
-*;q:
Discharge, Q = 0.1025 m3/min = 1'02.5 lit/min

A weigh tank receive s 7 .65 kg of liquid having sp. gI. of 0.85 in 14.9 seconds.
H1 What is the flow rate in liters per minute?
fe^ au
f= I " Eq.6 -52 Solution
u
JQ,
t t2
flow rate,M= = 0.5134 kgls
Mass
ffi
lf the flow is through a suppressed rectangular weir: Mass flow rate, M= PQ

,=f.",ffi 0.5134=(1000x0.86)Q

:i: fl ;,82,i,/min
8=Z;?i;
,=#l,:"*=# L"-'. ),, F r-lH:

Problem 6 - 3
Calculate the discharge in liters per second through a 100-mm diameter orifice
under a head of 5.5 m of water. Assume C" = 0.61' and G = 0'98'
.2A.lt1l
'=*lWm] 'q5'53 Sohrtion
Q= cAJzsH
where G, = weir factor
C = C" x G = 0.61 x 0'98 = 0.5978
L = crest length
H=5.5m
A, = constant water surface are of reservoir or tank l:-
Hr = initial head e = 0.Se7s t (0.100), J2(e.81)(5.s)
H2 = final head Q= 0.04877 s13/s = 48.77ls
3oB F,:f,;tl'^flasurernent FLUID MECHANICS
FLUID MECHANICS
& HYDRAUL'CS & HYDRAULICS F,uidFrow;lf:15*,:f 3oq
Problem 6 - 4
An orificq has a coefficrpnt of.discharge Solution
of 0.62 anda coefficient of .contraction
of 0.d3. Determine the coefficien, "^
;f ;:i;; io, tt
" ".iii"".---." Q= CA.tzg
Solution
o'82
C=C,x(,, H=1.+2.5*
r'r +1.5 1.5
0.52= 0.G3 x C,,
H = 3.487 m of glycerin
C, = 0.9sa
Q = 0.65 x f; (0.125)2
" J2gS1\t487)
Q = 0.056 m7s

Ca.lculate the discharee through


flre 140-mm diameter orifice shown. Assume
C = 0.62.

,A

The discharge through a 75-mm diameter orifice at the bottom of a large tank
was measured be 1.,734liters in L minute. If the head over the orifice remain
constant at 5.5 m, compute the coefficient of discharge.

Solution
-_ a
Solution er
Since the head is constan! the flow is steady, thus;
Q= CArpA _ Vol 1734/1000
H = HUp.t unn, - HDownstreanr
Q=u*;= 1i6o)-
H=3+ 50 15 Q = 0.0289 rrrl/s
9.81 9.81
er =.A JTfr = {- (o.o75r 1E(es1)(ss)
H = 6.568 m
Q = 0.62 x (0.14)z
Qr = 0.04589 m3/s
nz f(r3-1)Gs68) _ 0.0289
-
Q = 0.L08 m3/s '- o.o458g
C = 0.53

An open cyrindricar tat*' 2'4m in Problem 6 - 8 (CE May 1991)


diameter and
= 1.5), 2.5 m of water,.and 1.s-,. orrir !-T t{r has 1 m of grycerin (s*
the 125 mm diameter located
i;;l osz)-;;;"r*il"ti", dir.h*g" A calibration test of a 12.5-mm-diameter circular sharp-edged orifice in a
ua'u-,u uono* of the tank. vertical side of a large tank showed a discharge of 590 N of water in 81
[lJ;"n Assume c =
seconds at a constant head of 410 m. Measurement of the jet showed that it
kaveled 2.35 m horizontally while dropping 300-mm. compute the three
orifice coefficients.
3 I O F,Xfi'J,Il'ilasurement FLUID MECHANICS CHAPTERSTX ?l I
& HYDRAULICS Fluid Flow Measurement J I I
Solution
Theoretical values: Conkaction, C:
C= CrCu
o,= Jifr
= JN.B\@fi 0.631 = C,x 0.989
q = 9.603 rn/s C. = 0.638
Qt = Aoy = i Q.01.25)rx (9.503)
Q' = 0.007178rfi/s
Problem 6 - 9
Actual values: A 50-mm diameter circular sharp-edged orifice at the side of a tank discharges
Actual discharge: water under a head of 3 m. If the coefficient of conhaction C. = O.eS and the
_ Vol head lost is 240 mm, Compute the discharge and the coefficients of velocity C,
Q= ti^" (steady flow) and discharge C.

vor= If = g
y 9810 Solution
Vol = 0.0601 m3 Energy eq. between 1 and 2:
_ 0.0601 Er-HL= Ez
v- ,t _-
81 ar2 +?7 ayr-HL=azz +lL+s,
Q = 0.000243 rr:f / s 29yZgy
Actual velocity: a2
,
0+0+3-0.24= +0+0
x=2.35m ,8
0=0o'
/=-0'3m
"
,12
= 2.75 rn
2g
8i' s )
'v=xtane-2t'2 cos2 o o= 7 .359 m/ actual or real velocity

9's-1(2.3-5)2 Theoretical velocity:


-0.3 = 2.35 tan o" -
zoz cos2 0" ,,= ,[S = ilEE81X3)
o = 9.502m/ s u1 = 7.672 mf s ) theoretical or ideal velocity

Coefficientof velocity, C,,= L


Coefficients:
o
Velocity, Co = at \
Coefficient of velocity, ,= #.= 0.959
Velocity, g,= 2& = 0.989 Or, using Eq.5 - 5:
9.603

g ru=(L-t\Lzs
Discharge, C=
et [c,' )
Discharge, .- ,= 29^9^9':'= =0.G31
0.001178
"r=(+-t)ezet=oz+
3I 2 rtXffJ,:ltffiasurement FLUID MECHANICS CHAPTERSTX ata
& HYDRAULICS Fluid Flow Measuremenl J I J
1
;2 - 1 = 0.08696 Solution
Lz,
C, = 0.959 ) coefficient of velocity When the orifice is opened, the
bargewill sink a volume equal to
C=C.xC,,=0.63x0.959
the volume of water inside the 0.5
C= 0.6(X ) coefficient of discharge barge. Since the cross-sectional E
u1
area of the barge is constant and its
thickness is negligible, the barge
will sink to a depth equal to the
An orffice of 50 mm square, with C = 0.6 is depth of water that goes in. Thus
pak=-0.3mHg
located on one side of a closed cyfinaricaf the head over the orifice, being
tank as shown. An ' open 'rr"..rry submerged, is kept constant at L.5
manometer indicates a pressure head
of _ m.
?.09
* Hg in the air at the top of the tank.
If the upper 4 m of the tank is oil (sp. will sink to its top
The barge
gr.
0.80) and the remainder is " wlter,= whenx=0.5m.
determine the discharge through th;
orifice. Volume = Q f
Solution Volume = 5(10X0.S)

Q= califr Volume = 25 m3

1_=
4* a(0.8) + (-0.3X13.6) o= cAJrsH
H = 3.72 m of water
Q = 0.6 f (0.18)11E(es1)(15) = 0.08283 m3/s
Q = 0.5 x (0.05)z /{t81)Fl2) 25 = 0.08283 t
Q = 0.01173 ms/s f = 301.83 sec = 5.03 min

Problem 6 - 12
A steer barge, rectangular in pran, floats Calculate the discharge through a 90-mm-diameter
with a draft of r..5 m. If the barge is L0
m long, 5 m wide, and 2 m d""p, sharp edged orifice in the figure shown. Assume C
Ume *""rr*y-,"']rr,t ,r ro U,
top edge after opening a standard "ornprte-the
orifice, 1g0 mrn in diameter, in its bottom. = 0.65
Neglect the thickness of the verricat ,il;;;; j]:-'*" "' '
"*;;; Solution
Q= cA^lr@
H=3+9.81x24
0.90
H=5,71,8m
Q = 0.65 x f, (o.o9o),' \@.7r8)
Q = 0.0438 63/s = 43.8 Vs
3,t 4 F,XtrJ,:l',nasurement FLUID
&
FTUID MECHANICS
& HYDRAULICS F,uidF,owfi:*ilfl:X 315
Problem 5 - 13 (CE May 2001)
water flows through an orifice at the vertical side of a Iarge
tank under,
6.oe = o.eS JW
constant head of 2.4 m. How far horizontaly from H = 1.968 m
the vena ."o"ou.,u *iu *
jet shikes the ground 1.5 m below the orificei
H=1.+s=1.968
s = 0.968
Solution
o ,'2
y=rtane- ---+- ;
2on2.or16-
'y=-1.5m A large closed cylindrical steel tank 4 m high with its bottom on a level
u,= ,[zgn grooni contains layers of liquid. The bottom layer is water 2 meters deep'
traro
irr" top layer is occupied by a liquid whose specific gravity-is not-known, to a
a,: JN.8\C4 depth of i meter. Tire air-space at the top is pressurized- to-16. kPa above
ao= 6.862m/s atmosphere. A 50-mm-diu*Lbr orifice with a coefficient of velocity of 0.98 is
0=0o situated one meter from the bottom of the tank. The jet from the orifice hits the
-1.5=xtanoo- 9'8't'x2 ground 3.5 m horizontally away from the vena contracta. Deterrnine the
2(6.5642 coo2 specific gravity of the liquid at the top layer'
0o
x=3.79m
Solution
From the trajectorY:
n*2
A large cylindrical rt::l u=xtan0- -#---'=-
11t_4 m high with its bottom on a level ground " zuo' cos" 0
contains two layers of riquid. The bottom rayer is water
2 meters deep. The top 0=0'
Iayer is occupied by a liquid *h::.u specific graviry
1 meter. A 50 mm diameter orifice with a-coefficient
i, ; k;;;;; a depth of 9.81(3.5)2
situated one meter from the bottom of the tank. The jet
of verocity of 0.9g is -1 =0- ----:--4:-
r."* n" ,rince hits the n' cost 0'
2u
ground 2.75 m horizontaily away from the vena conkacta. ao=715m/s
Determine the
specific gravity of the liquid at thelop hyer.

Solution u"= C,r[TsH

oo= C, ,lrg4 7.zs = o.ss J@6H


H=1+11s; H=3.19m
H=1.+s H=L+rG)* ffi =a.re
FromY=rtan0- {*'^ s = 0.55
2un2 cosz e

9.81(2.7s)7 2m
-1= 2.75 tan 0o -
2oo2 cosz oo
oo= 6.09 m/s
r? It vt-
CHAPTER StX
Fluid FlowMeasurement FI.UID MECHANTCS ]r
FLUID MECHANICS
& HYoRRur-lcs & HYDRAULICS HuiaHowfiH:LT:X 317
Problem 6 - 16 (CE Board)
A jet is issued from the side of a tank under (c) Velocity of the jet as it strikes the ground:
a constant head of 3 m. The side ,
of the tank has an incrination of 1H to 1v.
Th;
is 6.70 m. Neglecting air resistance and assuming
i;;il;il;
*"i"i," the tank ,
Work-energy equation between O and 2:
C,, = 1.0, determine the KEs+WYr=YB,
following:
(r) the maximum height to which the jet
,/rW ao2+Wh=ryrL or,
will rise, 88
(b) the-point it strike a horizontal plane 1.20
m below the bottom of the
tank, and L5'*-+4.9=az2
k) the velocity of the jet as it strike the ground. 2(e.81) 29
u2= 12.45 m/s
Solution

Problem 6 - L7
Determine the diameter of an orifice that permits a tank of horizontal cross-
section 1.5 m2 to have its Iiquid surface draw down at the rate of 160 mm/s for
a 3.35-m head on the orifice. Use C = 0.63.

Solution
The discharge through the orifice is equal to the tank's cross-sectional
area times the draw down rate.
Q = At"nt X ?draw down

Q=1.5x0.15=0.24rfi/o
a"= C,$F
?"= (1) Jm-tm =7.672m/s IQ= CA",{zgH I
(a) Maximum height (at point 1,ay= 0.24=0.63" 2Or,{ifi$
0)
From physics,
D=0.245m=245mm
uv2= aoyz - Zgy
g = (7.572 sin 45o)2
- 2(9.81) yr
/r = 1.5 m ) maximum height above the orifice. Problem 6 - 18
(b) Point is-skike the A 7S-mm-diameter orifice discharges 1.812 m3 of liquid (sp. g. = 7.0n in 82.2
ground (at point 2, yz= _4.9 m)
From physics: seconds under a 2.75 m head. The vqlocity at thgvena contracta is determined
by Pitot static tube with a coefficient of 1.0. The manometer liquid is acetylene
A= xt;ane-
' 8" tetrabromide having a sp. gr. of 2.96 and the gage difference is 1.02.
2ar2 cos2= 0 Determine the three orifice coefficients.
t

-4.9 = xztan 45o - e.8\)x22 solution


45o The actual velocity of flow using Pitot static tube is given by:
0.167 x2z - xz - 4.9100*"'cos2
rz = 9.18 m ) horizontal distancefrom the oriiice
o
r? tr 1, CHAPTERSTX
FLUID MECHANICS CHAPTERSTX 2to
Fluid Flow Measurement
& HYDR & HYDRAULICS Fluid Flow Measurement r ' '
where:
Coefficient of contraction:
C, = Pitot tube coefficient
R = gage reading [C=C.xGl
0.6792=(.x0.809
so = sp. gr. of the gage liquid
C. = 0.8396
s = sp. gr. of the liquid
flowing

a=Cr
trFl Problem 6 - 19 (CE November 1999)
A closed cylindrical tank 5 m high contains 2.5 m of water. A 100-mm circular
orifice is situated 0.5 m from its bottom. What air pressure must be
u=1.0 zs(tozt(ffi-l maintained in the air space in order to discharge water at 10 hp.

a = 5.9455 m/s Solution


Theoretical velocity through Power = Qt E
the orifice:
a2
a,= J@ =
q=7.3454m/s
Jze.Bl)e.A Power = (Aa) y
6 1

L0(746)= f (0.1)' (e810) -l--


Coefficientof velocity, Cr= 3- o = 12.38 m/ s
?t
Coefficient of velocity, 5 u= Jzgu =12.38
C,, = ?115
Coefficient of velocity, ,, = {.#;n H=7.82m 0.5 m

Theoretical discharge: P =7.82


H=2+9.81
Q,= A,,flfr = ut
(O.O7S)r,!T@eB P = 57.09kPa
Qt = 0.03245 mt / s

Actual discharge: Problem 6 - 20


..., _ volume 1..g1,2 A concrete culvert 1.2 m in diameter and 5 m long conveys flood water. Both
time 92.2 ends of the culveri are submerged and the difference in water level upstream
Q = 0.02204 m3/ s and downsheam is 2.40 m. Calculate the discharge assuming C = 0.61.

Solution
Coefficientof discharge, C
= I cA^pn
Qt Q=
Coefficient of discharse . g 0'02204
'
0.0324s
= Q=0.6Lx f (1.2)2 .81X2.4)

Coefficient of discharge, C = 0.6292 Q= 4.734m3/s


3ZO Filf,T:l'ffiasurement FLUID
FLUID MECHANICS CHAPTERSIX -^.
&l & HYDRAULICS FluidFlowMeasurement 5Z I
Problem 6 - 21
lt is desired to divert 5'1 m3/s water from a pool whose Theoretical discharge, Qr = 0.033 6o/s = 33 L/ s
water surfr
;f::T,lrT"l :" aaiac"nt p""J *r,"r" water surface etevation is 42 ^ 23.4'l
by means of a short..:concrete
What size of culvert is need.ed
culvert s * 1"";;;;;r"dil[ :iT:; 33
*r"*i"* il'8 SSZ C= 0.709

Solution Coefficient of contraction:


C= C,x Co
0.709=C.x0.78
C, = 0.909

Q= ca,lrF Problem 6 - 23
H=45-42=3m Culve(, D = ? A standard short tube L00 in diameter discharges water under a head of 4.95
m. A small hole, tapped in the side of the tube 50 mm from the entrance, is'
5.1 = 0.58
" t @), J4e.81)(3) connected with the upper end of the piezometer tube the lower end of which
D ="1..2't m is submerged in a pan of mercury. Neglecting vapor pressure, to what height
will the mercury rise iir the tube? Also determine the absolute pressure at the
upper end of the piezometer tube.

A 7,-mm-diameter orifi,ce discharges Solution


23.41 liters per second of riquid
head of 2.85 m. rh" urder a Note: For standard short tubes, the
liq*:,:;;f;;";;;il;;;;;ilT
callipering to be 66.25 mm. Carculut"
*rl *,ru" orifice coefficients.
found by pressure head at point near the entrance
is -0.82H. See page 277.
Solution
Coefficient of contraction: lL = -o.szH
Vr-
ad" I
-A-.
-- D2 Po =-0.82(4.95)=4.0u,
66.252 v
U. - -------=-
p, = '4.059(9.81\
75.
C, = 0.78
pn= -39.82kPa

Coefficient of discharge: Neglecting vapor pressure:


?-a ,-_Po _ 39.82
er nt 9.81,x13.6
nl

Actual discharge, h = 0.298


e = 23.41 L/ s ft=298mm
Theoretical discharge,
er= A rl|{![
Theoretical discharge,
e, = t (0.025),
iE(9-8 U(2.8,
322 c},APrERs,x
d F tow ieasurernent
F I ui
FLUID
Probtem FLUID MECHANICS CHAPTERSIX r-r
6 _ 24 &
& HYDRAUUCS FluidFtowMeasurement 525

Solution

Solution

"".'iti.'f "^];';i:1;;s. trre coerficients ora Borda,s mouthpiece are _


e 0.
Or1lr}l. vena
conrracta:
a=C,A3t
Qr= Qz= Q
i*=o.to*
a = 106.L mm
f,(tso1z

Discharge:
Energy equation between 1 and 2 neglecling head lost (theoretical):
Q= CArl@
Er=Ez
C=C.xC,
C= 0.5 x 1.0
yr' *P, *r.=rz2 ! !-L
D.
122
= 0.50 29v29 v

3:i:1,-;{,is)'1EF}m =8Qr' , +lL*g= =8Qr'-, *g


n2g(0.30)a Y n2 g(0.075\a

Pt e Eq. (1)
26otqs =
v

Sum-up pressure head from 3 to 1 in meters of oil:


19- + 0.1sffi -0.7s=
#;:,*::;;;'#'"ffi:ffi: 75mmo +
;;:L":::"'r ."i*oll ;H:
1"1i":":',",:l.y;;;;;f "i? lL = 2.13m of oil
j;"11,f ",in",,rL"',iX,X.,.[j
v

il*",T."ilT:€;
velocityur ,f,"i"i. # ::il:; In Eq. (1):
2601Q.f =213
Qr = 0.0285 )
Theoretical discharge (since HL is neglected)

Actual discharge, Q= CQr= C, Q,Qr


Actual discharge, Q= 0.62 x 0.98 x 0.0286
Actual discharge, Q= 0.017qm3/s = 17.4lJs
324 F,Xffi:1:,f;asurement FLUID MECFIANICS, FLUID MECHANICS CHAPTER StX
arE
& HYDRAULICS & HYDRAULICS Fluid Flow Measuremenl Ja'J
Actual diameter of the j€t, d:
Problem 6 - 26
gr=o =L oil flows through a pipe as shown in the figure. Determine the discharge of
ADz oil in the pipe assuming C = 0.63.
:2
d
.0.52=
7sz
d = 59.1 mm

Actual velocity:
Theoretical velocity, ,,
-A = QL
' Theoreticalvelocity, ,r= =6.474m/s
Acfualvelocity, a=Cour
ffi
Actual velocity, a = 0.9g(G.474) Solution
Actual velocity, a = 6.544m/s . Qt= Qz= Q
Another Solution: Energy equation between 1 and 2 neglecting head lost (theoretical):
The Discharge through this type of orifice Et=Ez
is given by:
rrr2 Pt +21=a: +lL1s,
29Y
-?-
z8y
H
8Q12
=rQr' , + lL +0,= =rQr' *
g(0.250)a y
p,
12 n2g(0.100)a

805 Q.F ='Pr - Pz ) Eq. (1)


Er where C = coefficient of discharge l
\t \t

rr Ao = dr€a of the orifice


Sum-up pressure head from 1 to 2 in meters of oil:
Bl Pr .
I,
tl = pressure head at L in meters or feet of the
fluid flowing Pr +z+0.35-0.3599 -7= P2
f Y -0.e1 y
ti D = diameter of orifice
lr D, = diametgr of pipe Pt - Pz = 4.881 m of oil
yv
C=C.x C,=0.62x0.98 In Eq. (1):
ll"
,1,, C = 0.5076
805 Qf = 4.881
Qr = 0.07787 mt/s
I

2(e.81X2.13)
Q=0.6076x f,e.aez 1- (0.607q2
(7s / soq4 Actual discharge, Q = C Qr = 0.63(0.07787)
Q=0.0174rfi/s=t7.4 Ue Actual discha.rge, Q= 0.0491.1n3/s = 49.!L/s
32 6 F,Xf,T,:l',Lsurernent FTUID MECHANICS
Fluid Flow
CHAPTER SIX
--a
Measurement 5Z I
& HYDRAULICS
The discharge through this orifice is
given by: Solution
Since the cross-sectional area
of the tank is constant, we can
tseBq.6 -27.

where C = coefficient
,=ffitJE;-l:r'l
of discharge
An = area of the orifice Hr=2.5m
Pt Hz=1.0m
- pressure head at 1 in meters or feet
of the fluid flowing
;:
Pz
= pressure head at 2 in meters or feet of
f the fluid flowing
100 mm O orifice
.
A, = area of pipe
Ao/Ap= (D"/Dp), ,- 2*t$'5)2 6tr=-"nr)
= 0.60

0.60 x f, (0.1)" x !'2(9.81)


t = 98.4 seconds
I zrPr -Pz
Q= CA"l
tl1-t;,
Y

/ A)2 Problem 6 - 28
A 100-mm-diameter orifice on the side of a tank 1.83 m in d.iameter, draws the
Q=0.63x f (0.1)z
surface down from 2.M m lo 1.22 m above the orifice'in 83.7 seconds.
Calculate the discharge coefficient?
Q = 0.0491mt/3,
Q = 49.1, Qs Sotution
Since the head vary, the flow is unsteady.

=?o'= W
'= LAoizg -Wl
A 1.5-m-diameter vertical cyrindricar ' 2i$.$2
tank 3 m high contains 2.5 m of water.
1,0-mm-diameter circular sharp-edguJ A
oriri." is rocated .at its
e.t(o1oofl@ Vm-lTEl
bottom. 83.7 =
Assume C = 0.60.
(a) it take to lower the water tevel ro 1 m
deep after opening C = 0.8255
H::r[::rwilt
(b) How long will it take to empty the tank?
Problem 6 - 29 (CE May 1999)
An open cylindrical tank 4 m in diameter and 10 m high contains 5 m of water
and 4 m of oil (sp. gr. = 0.8). Find the time to emPty the tank through a 100-
mm diameter orifice at the bottom. Assume C. = 0.9 and G = 0.98.
328 F,Xfi'J,:l'ffiasurernent FLUID MECHANICS
& HYDRAULICS
FLUID MECHANICS
Fluid Flow
CHAPTER SrX
?)a
Measurement Ja ,
& HYDRAULICS
Solution

,=4w46) f =? from Hr =

t=sJ62V10
-
I
1.0

-J4l
-l
to Hz= 1,0 _ 6=4

2A, 2t(4)2 t = 3.675 minutes


cAl[4 (o.e x o.es)f (o.i)2
J 2(eE
2A, Problem 6 - 31
= 819.1'
cA{2s The initial head on an orifice was 9 m and when the flow was terminated the
head was measured at 4 m. Under what constant head H would ttre same
Time to empty the water: orifice discharge the same volume of water in the same interval of time?
H't=6+4(0.5)=9.2
Hz = a(0.8) = 3.2 m Solution
Under variable head:
rr = 8re.1 (,[d2 -,|tZ)
fr = 1019.3 sec ,=ffi(lr;-.m)
Time to empty the oil:
Hr= 4
t= 'o=
CA",,l2g
tJg - J+l
Hz= 0
l- _\ 2A,
tz=819.1,(J4 -J0) t=
tz = 1,638.2 sec
CA,,l2g

Total time to empty = h + tz= 2657.5 sec - head:


Under constant
Total time to empty = 44.3 min Volume = Q f
A"(g-4)=CA",pgH
ffi
A tank circular in cross-section-is r.0 m high. 5=2JH
It takes r.0 minutes to empty it H=5.25rlr
through a hole at the bottom when the
fuil of water
rr sslr qr uts start. How
at the
long will it take to drop the upper o ," "r;'t-i,
or *ui",
'
Solution Problem 6 - 32 (CE Board 1991)
2A' l^m-wl
1=c A,rl28 A vertical cylindrical tank has an orffice for its outlet. When the water surface
N in the tank is 5 m above the orifice, the surface can be lowered'4 m in 20
minutes. What uniform air pressure must be applied at the surface if the same
f= r(k4-JH,l volume of water is to be discharged in 10 minutes?
f = L0 min from Hr = ]0 m to H2 = g 1n

10=K[.m.-,fi']
K=3.L62
330 F,Xfi'J:l'ilasurement FTUID
&
MECHANICS
Fluid Flow
CHAPTERSIX 221
Measurement J r '
HYDRAULICS
Solution
6 - 33 (CE 1ee2)
,=#W-ml A composite non-prismatic S-m.high
pllndrical tank shown has a frustum of a
2A' i36ne at the bottom with upper base
let K = dlameter 2.5 m, 1.25-m-diameter at the
CA"J4
bottom, and 2 m high. The bottom
t=KtJHt-Jr,l Esntains 100-mm-diameter sharp edged
orifice withrcoefficient of discharge of 0.60.
In Figure (a): lf fully filled to the top, determine the time
t=20min to empty the tank in minutes.
Hr=5m; Hz=1m
r= rffi -"fl;l Solution
The cross sectional area from
zo = rfus -;f] level L to level 2 is constant.
K = 16.18

In Figure (b):
,,=ffitla--p;l
f=10min A,= t (2.S;z= 4.91.rn2
bt c= fu- CA,Jzg = 0.6 x t Q.\, Jzg
v I
100 mm o orifice
.Hr=5 Po
cA"Jzg = o.o2o87
* =5*, C = 0.60
Hr=9m; Hz=4m
v
-lI

H2=l+ Po =1*,
v
n=
mF-A] =47o.5sec

| Figure (b)
t=KILJHl _Jnr) Ume = 10 min From level 2 to level 3:
- -r
T- A, dh
1o=16.18t",6+c-.ffiJ ,=
J5+c =0.618+ ,lTli r, CA",lLgH r 1.25

A"= nta | 0.625 0.625 |

Squaring both sides: r-0.625+x


5+c=0.3819+7.,*,6JTi +1+c x 0.625 o.6?s ,-
i= 4 ,*= n "
lffi
.JT* =2.e273
r=0.625* gPh= o.(85 (4+,,)
Squaring both sides:
1+c=8.559 A"= nlff (4+h)lz
A,=0.0767(16+8h+h2)
c =7.569 m = Po H=lr
v
p" = 7.569(9.81) 0.0767(15+8h+h2\dh
pn = 74.25kPa
fr=
f CA,J*Jh
332 F,:f,'J;1'ffiasurement FLU'D MECHANICS FLUID MECHANICS CHAPTERSTX 222
& & HYDRAULICS Fluid Flow Measuremenl JJJ
007fr!1-o!-a!-*=t,'la,
,,= =3.615 +8tr1/2 *t,r/rht,
Hr=3, 5rm; Hz= 0
t^ 16r,,.,, .3.5
+7h+ #\dtt
0.1.M(L2.25
t- = 480
= z.azSfto{z1ht / 2 + 8 nz r z +
t4
Jo o.62AoJ4es1)J'
! ! tts/ z
l,
= 3.675[321+1r r z * g13 / 2 2]
! * rz 1+1s
r

tz = 439.04 sec
^ r3.5
g1,54.2A" = l rU",,, *
14
t,u /, * !trut'
Total time to empty, t = t1 + 1, L3 5 I

_Jo
Total tirne to empty, t = 470.5 + 4gg.04
Total time to empty, t = 909.S4sec = 15.L6 minutes s154.2A" =lzsP.sY t z + r2 *
ff Q.il3 f te.sluz']- o
A.=0.0094486m2= z-D2
D=0.1097m=109.7mm
A tank in the form of a frustum of
right circular cone 1.50 m in diameter and
a
the bottom, 3-m-diameter at the top, aid 3.5 m
high, is fun of water. A sharp-
edged orifice with C = 0.62 is rocated at the bottom Problem 6 - 35
of the tank. what diameter
of orifice is needed to einpty the tank in eight A sharp-edged orifice 100-mm in diameter, in the side of a tank having
minutes? a
horizontal cross-section 2 m square, discharges water under a constant head.
Solution The rate of inflow over which the head was kept constant is suddenly changed
from 20 lit/sec to 30 lit/s. How long will it take, after this change occurs, for
the head to become 2 meters? The coefficient of discharge may be considered
,=
[,'# constant and equal to 0.60.

Solution
Q"= CA",lTfi
H=h
Q" = 0.62A",pfi Q.* = CA
^PA
A' = nP2 = 0.60 x +(o.loYJzpfI,
R = 0.75 +: Q"t= 0.02087 hr/z
1 = 0.75'
h 3.5 Solving for /r1:
*= wh Since the head was kept constant
when Qi,, = 20L/s, therefore
R=0.75*Wh
Qo,t=20L/s=0.02mt/s
R = gi5 (3.s + r,) Q",1=0.02097 hf/z = g.g2
A. = rr[rqf e5 + t41z Ia = 0.918 m

A, = 0.144(12.25 + 7h + h2)
f=8min=480sec
? ?a. CHAPTER StX
r- t Fluid Flow Measurement F[UID FLUID MECHANICS CHAPTERSIX 22E
& & HYDRAULICS Fluid Flow Measuremenl J J J
When the in-{Iow was suddenly changed
to 30 L/s: Problem 6 - 36
. f" A-dh The rectangular tank shown is divided by a partition into two chambers and
Ju, Q,, -Qn,, provided with a round 1"50-mm-diameter sharp-edged orifice at the lower
Q," = 0.03 m:/s portion of the partition. At a certain instant, the level in chamber B is 3 m
Qo* = 0.02087 ll/2 higher than it is in chamber A. How long will it take for the water surfaces in
A,=2x2=4m2 the two chambers to be at the same level? Assume C= 0.62.
Hr = ht = 0.918 m
Hz= lb= 2^

t= f
12
4dh = l2 4dtr
Jo.er8 o.o3 -o.ozolzhl/, Jo.rrr@
a2
t=t91.66 1 dlt
)n.rrrt.+32 _ ll /z
' Let 1.437 - Jl/2 = a l

ll12=7.437_x
h=(7.437-x)z
dh = 2(1.43Z - x)(-dx) = _2(1.437 _
x)dx
When/r=0.918; x=0.429
I4/henft =Z;x=0.0?29

Solution
-'--
t = 1,97.6G ln',"' - ---;-
zl.qgz - *1a*
Jo.n, Ae
p0.0229 . /flffie
='383'32 (ry"-*) k_
Jrrr,
A,2dH2
= -383.32[1.4 37 tn x - ,=
-
]::;' I:, Qo

f = 1,500 sec = 25 minutes


Q= CA"JIyH
dH= dHt+ dHt
rH --
ldVl = dVzl
A'rdHt= A'zdHz
A'2
dH'' = A.r dH"
an=
ffan2+dHz'=[*.r)*
336 CHAPTER SIX
FluidFlow Measurement FLUID
& MECHANICS . CHAPTER SIX r s
Fluid Flow Measurement 55 I
AH = Au aH,
{.,.rA,,

dH'1=
v#;dH
,=
r,'^"*:# Yr- Vz
,= ---1-- f '' A. *(2)'v= +(3)'?(1)
cA,\f4 Jr, nu#tH-l/z dH ) Formura !'2.25m
When A,r and A,z are tlp+Y+1-5
constant: =5-1-2.25
,=;#;"fu lh= L.75m
[i,'-','0, Tank

= io,r,o7, __l-: lzurt, 1- ArtAr2


A,1r A,z ca,,[$- y'" '- fr, --2-W -1ru
J u, Arr + Ar2 CAo$g
1= -ArtArz 2 Hr =5m
A,t + Asz cAm W Aql ) Formura
Ag= t(2)'=nm'
Hr=initialhead=3m A,z= t(3)2=2.25nm2
Hz=finalhead=0m
As1 = cl'oss-sectionar area
cA,J2g = o.6o x i 10.201,,{z{o.at1
of tank 1 = 3 x
)=6 cA"Jzg = 0.0835
"";;;;;; .;;: o1z

r=ffi
.- ti;;''s-sectional
r= 15 m2
*r--fi-"Elq-rl
| = 305.91 seconds
, _ x(2.25n) 2
n+2.25n 0.0835
ks -.n*]
I = 47.57 seconds

li: Y:tq*lare cylindricat, tanks 1


and 2 havino ,o^*^,^-_ A vertical rectangular water tank is divided into two
respectively, connected with a 200-mm_a1ll9.
d'lT"ters ^-ll
2 m a,,a e m,l chambers whose
Itorizontal sections are 3 m2 and 5 m2, respcctively. The dividing wall is
provided with a 100 mm x 100 rirm square hole located 0.5 m from the bottom
T:i"li",Tf":;ff
water surface in tank : after opening the tube, nrrd whose coefficient of discharge is 0.60. Initially there is 5 m deep of water
2 to rise by 1 meter? ror *re
ln the smaller chamber and L m deep of water in the larger chamber. What is
"H:*fl:I;i}ffi'sT::1fr?",#i:i"#itri1? the difference in the water level in the two chambers after 2 ndnutes?
33g F,XtrJ:l'^[asurement FLUID MECHANICS
& HYDRAULICS Fruid Frow,fi:*il5fffi 339
Solution
1n AsrAs2 __ 1_lrn _m)
From the formula: &r=3m2 A.z= Art + Ar2 CAo.tZg

'=-#h^fuw-l6) =Ar1A,r CA""lzg


2 tq-W)
t't
Ht=h't=4n..
H2= h2= |
f=2min=120sec
I
5 m
Arz
*1

w-JT,7=##trp;-.m-,-l
4lt*1 cA,,[zs
126=
i(2
lu=1.32m
G@*iWUr-,rE) I =ffito-'-.P;l
2(2.25n)
o.zlxft(0.2)2,lrpfrL ln -al
-39 t = 79.35 seconds
In the figure showrL how
long does it take to raise
the water surface in the
--T---
I
tank by 2 meters? The A swimming pool L5 m long, 10 m wide, and 3 m deep at one end and 1.6 m
I
right side of the figure is a 4m on the other end is fitted with a drain pipe 200 mm in diameter at the lowest
large reservoir of constant
part the pool. Compute the time required to drain the full content of the pool
water surface elevation.
assuming C = 0.80.

tolution

3mA -T-T
Solution El
@l
il
' As= f;(t)z Ar 4z=* +E
lm
At= 2.25nmz Er
ql I
Ar2= @
'rJ
Hr=4m
Ht=2m mmO
= 0.80

Time from level 1 to level 2 (constant water surface area)

3ma
'=#w-'t-ru\
CHAPTER SIX
340 Fluid Flow Measurement MECHANICS
HYDRAULICS Fruid F,ow,;:ffI5fffi 341
CA,\f$ = 0.80 x i. e.ZO1z,12f
6- 4r (cE May 2003)
CA,J4 = 0.111 aluice gate flows into a
A,=10x15=150m2 tal channel as shown
Hr=3m tlre Figure. Determine the
Hz=1.4m through the gate per
r width when .y = 1.0 m
dr = 6 m. Assume that
'=?1i:?w-ml
fr = 1483.3 seconds
pressure distribution at
ions 1 and 2 to be

Time from level 2 to level


ic and neglect
3; Iosses in the chamel
A, dlt : coefficient of contraction
,r=
l,r'irr'CAo,,f 2glt = 0.85 and coefficient of
Au=1 0x
ocity C= 0.95.

x '1,5

-=_
h 1,.4 d2= C,x Y
x = 10.7'L4lt dz=0.85 xl =0.85m
A" = 10('10.71,41) = .t
07.t4lt
cA,"JTil = o.8o x + (o.zq, Energy equafion between
JTsh O and €) neglecting losses:
CA,tfril =0.7711tt/z
h1 = 1.4 [Hl = Hr]
,)
lq= g 01,-
+5= t'z- +g.g5
t _ !fl'n to7.t+trdt, 2g 2g
L2- ,,2_,,2
Jo /2
0.t1.-t h1 ", "t = 5.15
2g

= e55.2ZS
F7.4
uz2 - u-r2 = 101.043 + Eq. (1)
- Jo lti/2ah
tQ, = Qzl
(6 xl)u = (0.85 x 1)zr2
= s6s.zzsl ?h?/2f14
L3 J,
at = 0."1417a2

= 965.225t $.43/2 _ O) In Eq. (1):


tz= 1,056 seconds uzz- (0.1417u2)2 = 101.043
0.97993 azz = 101.043; az ='10.754 m/ s
Total time to empty, t = +
h tz @, u,, = C,, uz = 0.95(10.154) = 9.6467 m/ s
Actual velocity at
= 1493.3 +.j.066 Discharge = Azoo = (0.85 x 1\(9.6467) = 8.2 m3A per meter
Total time to empty, t 2549.3r";;;1,
= = 42.49minutes
CHAPTER StX
?a7 CHAPTER StX
FLUID FLUID MECHANICS Fluid Flow Measurement
343
- t- Fluid Flciw Measurement
& & HYDRAULICS
Problem 6 - 42 (CE Board) Solution
A horizontal 150 mm diameter pipe gradualy reduces its section to
sOl
subsequently enlargrng i"to rso mm section. irr" pr"rr**n-u
fllmeter,
150 -mm pipe at a point just before entering the reducing
,""hoi is 1401
* trujj *T.r",.tiol at the end of the riducer, the p[rr.rr" i,7O oP,
::S mm of head is lost between
600 the points where tt
cornpute the rate of flow of water through the pipe. "
prurrrr.o ;;" k,o
l

Solution
Qt= Qz= Q

(a) Energy equation between 1 and 2 neglecting head lost:


E-r=Ez
u't2 ,p7 *,.=,r' * !2 +22 DC
Pr = 140 kPa
,gY2g Y
Qt= Qz= Q

Energy equation between 1 and 2:


8Q2
=8Q' , *b*o
-T'" *o= n2g(0.15)a
*lL v
78(040a
Et - HL= Ez
161..2Q2 =
Pr - Pz ) Eq. (1)

* .
8Q2
+
*2,-HL=
* *
ff *,,
Dt -Dt
Solvine for !-t---:- --'.
Y

n'1e.sr;10.ts14 *
9.81.
*o-0.50= ..-???, *J9-
' esr *e
n21e.ar;1o.orf
Y

Sum-up pressure head from 2 to


1 in [reters of water:
13057Q2 7 6.5356
Q=O.\ZlSpa/s= 22.4us ? * o+ 0.375(13.6) - 0'375 - t = +
Y

Pt - Pz = 43ZS m
6-
A 150 mm diameter horizontal venturi meter.is instalred
in a
:
ln Pqo(1)
diameter water main. The deflection of mercury in the di{ferential ' = 4'725
n 1ff1,2.'Q2
connected from the inlet to the throat is 375 mm. (a) Determine
the discl e = O,iZtZm3/s (theoretical discharge)
neglecting head lost. (b) compute the discharge if the head
rost f.";;;
to the tfuoat is 300 mm of water, and (c) what ii the meter head lost:
coefficient? (lr) Energy eq. between 1 and 2 considering
Et-HL=Ez
Y-L*lL*2r-HL=u22 * Pz ---
ZgY2g Y

lQz *Pr -0.30+0= 8Q2 + Pz- +g


n2g(0.45)a n29(0.15)4.
Y
Y
344 F,:f,'J:[',[,surement MECHANICS CHAPTER SIX ?4E
r tr
Fluid Flow Measurement

= ,,
v,.r' o.ro ) Eq.-"(1)
161.2 Q2
-y= lL
_
Pz +0.75+y+0.36(13.6)-0.36
y-T
161..2 qz = 4.725 - 0.30
Pt - Pz = 5.2g6m of water
Q = 0.165t7 mt/s (actual discharge) v

(c) Meter coefficient, C In Eq. (1):


6= Qu.*uI - 0.1.557 '153 Q? = 5.285 - 0.75
- Q*"or"* ffi12 Qr= 0.1722rrt/ s ) (theoretical discharge)
C = 0.9G8
Actual discharge, Q = C Q,
= 0.68 x 0.1722
Actual discharge, Q = O.1171 m1/s
A verticar venturi meter, 150 mm
in diameter is connected to a 30( lost (use the actual discharge)
#enffi::l;"T"J:::::1
If the deflection of mercury in :f * :#;ii".,,,r* b the throa t being 250 Qr = Qz= 0.1171'
the ain"rur,aa ,il.:"#:ffifrffitr;:ff Energy equation between 1 and 2 considering head lost:
fl"#_S:;1;;L,:,iir":f ,:::"dJ;1",;;e*,r;;;;;;""shL"_"t
the:neter coefficienr is 0.68. oetur*ir*
"rr"
il r*J ;::1 l:trf.flffi ffJ h- HL=
at'' + Pt +
12
'unZ
Pz +Zz
zt_HL= J3_ I
Solution
29yZg v

t-,PZ
8(0.117\2 8(0.1171)2
. *lL*'_HL= 't
+ 0.75
Qt=Qr=g ,21o.sr11o.a1a v ,21o.sr11o.rs;a

t""JtI Eq. between 1 and 2 neglecting


head lost:
gy= Pt-Pz .,t
-2.098-0.75
Ll - E2
=5.286 -2.098 -0.75
t-p,
29 y *r,=022 *pz *-_
' zg'7-zz HL=2.438rn

-;P:..r *!L*s= = 8Q,2


r,(9.81)(0.3)4 y " ,l(%1X0.1O.
UsingEq.6-5:
' ( t^'\'l ,,
HL=[J--rllr-
+ lZ
v
+ o.z5 [c,.tal )l ]a
C=C=0.68
153 P,z = Pt - Pz - o.7s ) Eq. (1) P = Q.rr'/r aP/s
v
,r=- (
,,, -t:-r).[,-fs$l'l
-', [ o.e, J -8(01171)2,
n21e.ary1o.rsya
Sum-up pressure head from 2 to 1
[o.us, L^ -l
in meters of water:
HL=2.439m
346 CHAPTERS'X
Fluid Ftow Measurernent
MECHANICS
6.45
Probtem I HYDRAULICS FruidFrow,;I*il"'*:H l4l
Neglecting losses.
calculate th; Pt - ?z =Y.t-Y2
discharge throueh Y T
the Venturi *utl,
snown_ Also, from the figure:
z"* !1 = 0.25 + yz
!/t -,!/z= A'25 -z
Pr -Pz =0.2s-z
yy
In Eq. (1):
153.07Q2=0.25-z+z
Q= 0.040Em3/s = 40.41./s
Solution

I- A 37.5 mm Venturi meter (C = 0.957)is installed ,r, u


horizontal pipe carrying oil having specific gravity of 0.g52.
,U-**IJ-
If the recorded
flow in the meter was 1.5 liters per s".or,d, what could have been the

I
Yz
cleflection of water in the differential manometer connected between the inlet
and the throat?

Solution
tco 75mmO

Qt=Q.=g 'co
Energy eq. between
1 amd 2 neglecting
head lost:
E
Lt --F E2
-.2
u7
*Pt +21- --2 * P2
29y 2gy" --
.-=-
8Q2
Bez
+Pl +,- -----'- D^
n'g(0.3)a , +0
n, g(0.15)4 y
Actual discharge, Q = 1.5 L/s = 0.0015 m3/s
753.0tq2=Pt -P2 n. ---
+z)Eq'(1)
Y Y since the head lost is not known, the theoreticar discharge will be used.
Q=cQ,
ffiPt
rl:;; ::;ffiT fl:T J
;;,lil ile,ffi ofrny*a,",,
point in the manometer
0.0015 = 0.957
Qr = 0.001567 rfi /
er
s

-Vt+Yz= Pz
| v Qt = Qz= 0'001567
I n I j:f,J:[,',fr u,.,,"-",, FTUID
D MECHANICS

Energy equation between


&
HYDRAULICS Fruid F,ow,fi:*:Hfffi 349
1 & 2 neglecting head lost
Er=Ez (using e)
at2 * pt --2 37.5 mm @ 37.5 mrn O
_ j-
29 y "- z8y'* P) *r,
-_

* pl _0= 8t0.g015642 .+ p2
8(0.007567)2
n's(0.075)a +o
Fs(O 03rsf ;
+ +=o.oe618moroil
Sum-up pressure head
from 2 to 1 m meters of oil:
Pzh Q, = Qz = Qs = 0.0085 m3/s

i*,. ,#, -t,-y= ? Y


tlh-z= o.os€ = o.o5
2g - Eo'ooss)2-n2 g(o.o2s)a
0..1737h=ir"= o HLta= 0.764m
Yy
0.1797h = 0.09618
h=0.554m=554mm
HLz.t= o.zt
- 29 = o.2o
8lo'oos5)2.
n2 g(0.025)a
HLz-t= 3.057 m
Energy equation between 1 and 2:
oil (sp. gr. = 0.8) flows ar
at rho *-+^ of o F a..
il-HL=Ez
the rate ^c 8.5 Iite
air^"ltur'norrr"l rift ut2 + lL * 21
pipe as shown i",h" r;;:Xl T"i:i"y:i^ i.,"T*ffifiT: $:-Hif#il 2g't-2gy - HLt-;= '22 * P, * ,,
attached from the Fj+: ffia:JJ:,::HH"".1J:;.r,ff#__diamet*
" ""base-of the t";i;H;r:rr,ag 8(0.0085)2 D. .,+ lL *o
Calculate the defrection
of
to the uase JrtJ:rT"I --r------- . +'' + A-0.764=
8(0.0085)2
Selcury i" ;""n il," if the head lost from the inlet n'g(0.0375)" Y n2 g(0.0'25)a v
,':ffiH:;:;"tJ:,$ fl:f*lg;'"=;:H and rrom *," th,out io the ouret Pr Pz = 13.03 m of oil
vl
37.5 mm @
Sum-up pressure head from 2 to 1 in meters of oil:
tt'o-Y-
D *,ry-h1)+1','1om "- Pt
y T
't6ht= L -b =B.oz
yv
ht=0.874m=8L4mm

Energy equation between 1 and 3:


E1 - HLt-z- HLz4= Es
350 F,Xf,'J,:l'nasurement FLUID MECHANICS
& HYDRAULICS Fruid Frowfi:lT"'*:H 35 I
. *,. b Problem 6'49
- HLt-z- HL.z-t= + * 2, a flowing
* ? * A glass tube with a 90o bend is open at both ends. It is inserted into
strlam of oil (sp. gr. = 0.90) so that one of the opening is directed uPsheam
and the other'is directed upward. If the oil inside the tube is 50 mm higher
ffi.+.0-0.764_s.os7=ffi + P3 +o v than the surface outside, determine the velocity measured by the tube'
Pt - Pz = 3.g21 m of oil
yy Solution

Sum-up pressure head from 3 to 1 in meters of oil: , = ,{Zgh (theoretical velocitY)


'?.g-to)*r,,*t$ y=+
= €(r81n0osl
Pt - ll =S.szt u = O.99 m/s
16hr=
yy
lu= 0.239 m :239 mm

A 30-cm by 15-cm venturi meter is mounted on a verticar pipe with


the
upward. One hundred awenty-six (126) liters per second of'oit
1rp. gr.
flows through the pipe. The throat section is 15 cm above the ,pst
If C,= 9'957, whdt is the difference in pressure betr,.veerl the inlet
Ll* sectior
and the throat? A Pitot-static tube (C = 1.0) is used to measure air speeds. with water in the
differential manometer unj u gugu difference of 75 mm, calculate the air speed
Solution
using p"i, = 1'.15 kg/ m3.
The discharge through a vertical Venturi
meter is given by the formula:
Solution

Q= C,AzJ?g

oit
s = 0.80 v

0.126 = 0.e57 x + (o.ts), Jz8 #ffi-o'u ---+


I
1-(0.15 /wq4 h=15 75 mm

fir!6-ots _t I
T

= 1,.582
' Water

P1 - p2= 22kPa
352 .tlf,J:Itffiasurement FLU'D MECHANICS
FTUID MECHANICS CHAPTER SIX t F.
& HYDRAUT'CS & HYDRAULICS Fluid Flow Measurement 555
Energy equation betw<
-'en 1 and 2 neglecting head
E lost:
L.t -r - E2 Solution
at? Consider two points 1 and 2
+ !L'+ z1 =
a22 D^
+ !-!- + z^ as shown in the figure. Point
100
1 mm O
2g Y2g v I is the stagnation poinf
a2
+ !! +e==1 Pz lrence z4 = Q.
,8 Ty
u
-_2 Energy equation between 1 and 2
,g- =Pz
,
-Pt -
-=-mofair )Eq.(1) neglecting head lost:
Et=Ez
')
Sum-up pressure head from 11L2
1 to 2 in meters of air: +P, *rr=rzz +Pz +2,
* ,* o.ozs io-09 2gv29v
* - o.o7s - rr = P2

o' o*lL+g=azz
y2gy +lL +g oil, s = 0.827
?yy - =64.s8mof air
)
In Eq. (1): 29vv -Pz mofair
+=Pr )Eq.(1)
a2
;-
zg
= 64.58
Sum-up pressure head from 1 to 2 in meters of air:
a = 35.6 rn/ s ) theoretical velocity Pt +u+0.0g-0.0ge810Ig.827 -o= lL
y'Lzul
AcfuaI velocity, 7t = C at
ActuaI velocity, o = 185.6)
= 35.6 m/s
Pr - Pz = 54.006 m of air
vv
In Eq. (1):
Air (tu N/m:) is flowing
= 12 uz
_-2
- 54.005
through a system shown. If 100 mm A 2g
oil (sp. gr. = 0.827) shows a
a2= 32.55 mf s
deflection of g0 rrrm,
calculate the flow rate
neglecting head lost.
Flowrate,Q=Azaz
= f (0.05)'z(32.ss)
Flow rate, Q = 0.0639t m3/s = 63.91. Qs
80 mm

oil, s = 0.827
354 F,Xf;?,:l'ffiasurement FLUID MECHANICS
& HYDRAULICS
FLUID MECHANICS
Fluid Flow
CHAPTER SIX ?trtr
Measurement rr-
& HYDRAULICS
Problem 6 - 52
A Pitot.tube in the pipe in which air is flowing In Eq. (1)
is connected to a manometer _.2
containing water as shown_in the figure.
If the difference in water revers in the
u' =7L.44
manometer is 87.5 mm, what is the velocity 2g
-- "-"
r of i" tf.,u
flow ^'r u'L pife;;;;;
rrvs' q
tube coefficient, C, = 0.99? ar = 37.M m/ s (theoretical velocity)

Actual velocity:
a= CoxtJl
= 0.99 x37.44
a = 37.07 rn/s

A flow nozz\eis a device inserted into a pipe to measure the flow as shown in
the figure. If Az is the exit area, show that for incompressible flow,
T

o o -- Q=c-,"|L zr(P'-P'\
\?{r)
l,tr1er 1ery2

where Ca is the coefficient of discharge, which takes into account frictional


effects and is determined experimentally.
Solution

Enogy equation between 1 (stagnation point)


and 2 neglecting head rost:
Et=Ez
a7',^ , Pt
urZ at
29 y '' k'T-"'
-?E+r-=:4-_f__
--2
0+ Ft +O= ut
+ lL *g
v 2g 'v.
"r?. -Pr Pz
) Eq, (1)
2g '.v v

Sq"f,rp pressure head from 1 to 2 in meters of


water:
4v"L2
-n-o.os75e81o +o.o87s +y= lZ
v
P't,
''.-
Pz *77.44mo1at
v v
356 F,:f,'J,:l'ffiasurement TLUID MECHANICS
.I HYDRAULICS Fluid Flow
GHAPTER StX
aEa
Measuremenl J) I
Solution
Energy equation between 1 and 2 neglecting head lost:
lroblem 6 - 54
Et= Ez A Pitot tube being used to determine the velocity of flow of water in a closed
Conduit indicates a difference between water levels in the Pitot tube and in the
rz2
{.2g Pt ' 29* lzi *,.
y *r,= 'Lz plezometer of 60 mm. lAtrhat is the velocity of flow?

Solution
t-Pt
29v29 +o=a22 *p2 *o , = ,{28h
az2 , = 1ae31xo.o6)
zs_rr2
zg=p1,'- ? ) Eq' (1)
u = 1.085 m/s

tQ, = Q,I
AtUt = Azaz Problem 6 - 55
rn = (Az/ Aiaz ln the figure shown, pressure gauge A reads 75 kPa, while pressure gauge B
reads 82 kPa. Find the velocity of air assuming its unit weight to be 20 N/m3.
In Eq. (1): Use Cr = 0.92 and neglect compressibility effect. .

*zg -,oro')"' Pt Pz
'2.9= y - y

[1-(Az/Ai2t* =
+ ?

Q= Ca x Azaz
Solution
Q=Cara, I
\u@, / A)2
,,[ry') o= 9,",,[Tgh

ffiw)
,.- PB Pa

n=ar"I vv
, 82,000 75,000 =350mofair
n=
20

Note: This formula can arso be used for Hoizontar ventui , = o.oz.@stlssq
--
Meters.
a = 75,24m/s
2EG!
JJs
CHAPTER StX FLUID MECHANICS FLUID MECHANICS
Fluid Ftow Measurement & HYDTiAULICS & HYDRAULICS FruidFrow,;:,fl1"'*:H 359
Problem6-56 i Problem 6 - 57
Carbon tetrachloride having specific gravity of 1.5 is flowing through a pipe. A rectangular, sharp-crested weir 15 m long with end contractions suppressed
The differential gage attached to the pitot-static tube shows a 100-mm is 1.5 m high. Determine the discharge when the head is 300 mm.
deflection of mercury. Assuming C1 = L.00, find the velocity of flow.
Solution
Solution
Since the height of weir is large compared to the head H, the velocify head
of approach can be neglected.
Using Francis Formula:
Q=1.84LW/z
Q = \.Bq (1s)(0.3)Y'?
Q = 4.535 m7s

Problem 6 - 58
A rectangular, sharp-crested weir with end contractions is 1.4 m long' How
high should it be placed in a channel to maintain an uPstream depth of 2.35 m
for a flow of 400 liters/second?

Solution

a= C,, JV{r
,- * Ps-Pe
v
R(Sgugenria -SRrria )
-
sflria
_ 0.08(13.6-1.6)
1..6
Using Francis Forrnula:
h = A.6m
Q = 1.84Lgt/2
u='!.x &r.81X0O L=7.4-0.2H
a = 3.43m/s
0.40 = 1.84(1 .4 - 0.211)113/2

Solve for H by trial and error:


Try H = 0.3
1.84[1.4 - 0.2(0.3)](0.3)vz = 6.495 ^,0.4 (oK)
:

From the figure shown above:


i
P=d-H
t P = 2.35 - 0.3 = 2.05 m

t
360 Ff,f,"J;:1'ffiasurement FLUID MECHANICS CHAPTER SIX -t , t
& Fluid Flow Measurement 5(D I
Problem 6 - 59
During a test on a suppressed weir 900 mm high,
_2.4_m the
maintained constant 1t 300 mm. .In 3g seconds, 2S,g00
fit?rs of water
collected. What is the weir factor C*?

Solution

Q = C,, L[(H + lh)trz -,ror,r,

" o
1.0.125
ud=:
A 7.s(1.8\
L=2.4m
ar= 015 mf s
0'752
Q= C,,, L[(H + h,)t/z - 7r,t1z1
rr,,=
29 -
'o 2g
^= Volume
Q -G;- (since the flow is steady) h,= 6.Ort, o,
125 = 1.88(7.5) [(H + 0.0287)3
10. /2 - (0.0287)3 / 2)
=
#
p = e./g/g 1g;1t/s
=7s7.eL/s
H = 0.777 m

Height of weir, P = 1.8 - H


= 1..8 - 0.777
Velocity of approach, ,o =I - 0'7579
Height of weir, P = 1.023 m
A 2.4(1.2)
Velobity of approach, ao= 0.2G31.6 m/ s
,_uo2
'2929 _(0.2631q2 Problem 6 - 61
l)ctermine the flow over a suppressed weir 3 m long and 1.2 m high under a
h, = 0.00353 hcad of 900 mm. The weir factor C,, = 1,.91,. Consider velocity of approach.

Q = C,,,QA)[(0.3 + g.gg353)3/2 - (0.0035 g)3/2) = O.7SZ| Solution


C*=7.891

A suppresseg
::tu ,t,_l_r:.lt: ,r
open channel. The weir factor C, = S^r^"hTge
10.125 rnl/s ofwater onto an
1.88. TJ what f,"igtli ;yli.u
built, if the water behind the weir must not exceed
1.g0; a""pe ' * *"i, U"
162 F,:f,?:I'ffiasurement FTUID MECHANICS
& HYDRAULICS Fluid Flow
CHAPTERSIX t,-
Measurement 5OJ
Q= C,u L[(H + - p,ztz1
hu)t/z
a-oz Problem 6 - 62 (CE November 1995)
IQ/(3x2,1)]2
29 -
rto=
Find the width, in meters, of the channel at the back of a suppressed weir
29
using the following data:
h,= 0.001284e2
Head,H=28.5cm
Depth of water, d = 2.485 rn
It can be seen that the discharge
e varies with la.which in turn Discharge,Q=0.84m3/s
Using this formuta" ai""Uy-_""rd
Irrhg Iead t" uirr-* Consider velocity of approach and use Francis formula.

Solution
I[:l#J::Hff:lfmate verocitv or approach by sorving
the Q = 1.8E Ll(H + h,)t/z - 1,ttz1
Q = C*L Httz
Solving for L and /r, using the forrnula:
Q = 1.91(3X0 .90)s/2 = 4.892 m3 s
/ Q = 1.84L113/2
tu = 0.001284(4.8s2)2 0.84 = 1.84t(0.285)3/2
lh= 0.0307 m L=3m
1f
oo2 (z"z-+aql2
2g -
lo.sE/
r1u= = 0.000547 m
I"- 9 = 1.91(3)[(0-9 + 0.0ZOn3/2 _ (o.Bon3/2]
New Q = 5.7't4m3/s
29
0. 84 = 1. 84L [(0.285 + 0.000647)3 /2 _ (0.000 642\s / 2)

h, = 0_.001284(S.1ttyz L=3m
ft, = 0.03358
UsingEq.5-33:

I"* I =
NewQ=5.133mi/s
1.e1 (3)[(0.e + 0.03358)3/2 _ (0.033s8)3i2] e= C-rs+ [r..,(+)']
L

The discharge converges


at 5.133 m3/s -'-rE-
,^_3C*' _
z
3(1.84)2
zg
UsingEq.6-33: G = 0.2588 -l
.r
e= " L '\a)l
C,LH|11*.,fg)'l 0.84 = 1.84(L,to.rrupr, [, . o.r?8rf
.-H.)'
L
L=3m
c,=29;-30jl.)2
229 2 2g-
Ct = 0.2789 Problem 6 - 63

e = 1.e1(3)(o rr,,l,. r rr'rfl)'l The discharge from a 15O-mm-diameter orifice under a head of 3.05 m and
coefficient of discharge C = 0.60 flows into a rectangular channel and over a
rectangular suppressed weir. The channel is 1.83 m wide and the weir has
Q= 5.1,43 m3/s approximately
height P = 1.50 m and length L = 0.31 m. Determine the depth of water in the
channel. Use Francis formula and neglect velocity of approach.
^, ,
50+
CHAPTERSIX
Fluid Flow Measurement
FLUID MECHANIC$ FLUID MECHANICS
& HYDRAULICS F,uid F,owfiHlX*:H 365
& HYDRAULICS

Solution Variation in depth = Hz- Hr = 60 mm


The discharge through the orifice equals the discharge through the weir. 0.574 0.481
N2/s N2/s
For the orifice: o'9??
= o ou
e = cA" JzgH = 0.60 x + (0.15)1 lE(es1)(3^0O N'r '
Q = 0.08202 m3ls N=3

For the weir (neglecting u,)


Q = 1.84LL1t/2
. 0.08202 = 1.84(0.31)H3/2 The discharge over a trapezoidal weir is 1.315 m3/s. The crest length is 2 m
H = 0.274rn and the sides are inclined at75'57' 49" withthe horizontal. Find the head on
tl're weir in meters.
Depth of water upstream of the weir:
d=H+P =0.274+1.50 Solution
d=1.774m The side inclination angle given is that for a Cipolletti weir.
Q=1.859LH3/2
1.315 = 1..859(2)I{3/2
Problem 6 - 64 H=0.50m
The flow in a rectangular channel varies from 225 liters per second to 350 liters
per second, and it is desired to regulate the depth by installing standard 90-
degree V-notch weir at the end. How many weirs are needed to regulate the
vatiations in depth to 60 mm? a spillway conkols a reservoir 4.6 hectares in area. The permanent crest is at
elevation 75 m. If water can be drawn from elevation76.5 m to elevation 75'5
Solution m in 42 minutes, find the length of the spillway in meters. Use Francis
For standard 90' V-notch weir, C,,, x L.4 formula neglecting velocity of approach.
Q = 1.Als/2
Solution
Let N be the required number of weirs
Total flow, Qr = N x Q = l.!l'llp/z

When the discharge is 0.225 m3 /s


0.225 = 7.4lt1grstz

= (head when the discharg eis225L/s)


", #+
When the discharge is 0.35 m3/s
0.35 = 1.41{H25/2

Hr= Y (head when the discharge is 350 L/s)


Nz/r
-),,
5oO
CHAPTERSIX FLUID MECHANICS CHAPTERSIX
^,q I
Fluid Flow Measuremey't & HYDRAULIC$ & HYDRAULICS Fluid Flow Measurement 50
UsingEq.6-53: Lengthofweir,L=1m
Initial head, Hr = 1 m
-t=2A,fr_rl
C,uL
LJ", Jrr,] The drop of water level after discharging 72m3 is
72
A, = 4.6hectares = 46,000 frz-
f = 42 minute s = 2520s".?tds
Drop
^ = 20(20) = 0.18 m

Hr=1.5m Final head, Hz= 1. - 0.18 = 0.82 m


Hz = 0.50 m Weir factor (Francis), C,,= 1.84
Cr,, = 1.84 (Francis Por4rula)

2(4oo)
- tg 1l
2520=
L = 11.85 m
#-t* h] t
1.84(1)
-
LJo.sz Jr l
= 45.35 seconds

Problem 6 - 68 (CE November 1991)


A V-notch weir is located or cut at one end of a tank having a horizontal
square section L0 m by 10 m. If the initial head on the weir is 1.20 m and it
A rectangular suppressed weir of lefl9th 1- m is constructed or cut at the top of takes 375 seconds to discharge 100 m3 of water, what could have been the
a tall reclangulariank having ahoftlontal section 20 mby 20 m. If the iniual
vertex angle of the weir. Use C = 0.60.
head over the weir is 1 m, compute the time required to discharge 72 cu. m. of
water; Use Francis formula. Solution
20m
Solution

./ .' .i-_
i '----
,-" '4-2.?a

When 100 m3 is discharged,from the tank, the water level drops by y


100xY=1gg
,-zerl I:-
t 1, U =1m
c*L LJH2 JH, Thus, the flow is unsteady with initial head Hr = 1.20 m and final head
Hz= 0.2m.
Water surface area at ury tim+ A, = 20(20) = 400 m2;
369 F,Xf,'J:l'ffiasurement FLUID
MECHANICS
&
IIYDRAULTCS Fruid F,ow,fi!,fil"T:I 369
t= lH'a,au Eonsider the horizontal strip shown (treated as an orifice under head /r)
Jr, Qnut ilQ = rIA,lzgh
A,=10x10=100m2
dA = 2x dlt
Qo,t= fi cJ2s tan$Hsrz= $1o.ao; ,f$ tanguun Express r in terms of lrby squared property of parabola:
Qo"t= 1.4'17 tan$ Hstz
*'
t = 375 seconds "t.s-h=!2
y = s.797 JLs-h
375 = S''20 rcoau
7.417
)s.2s tan$Hs/z dA = 2(0.707 Jts-n )an
p7.2O
(tQ= 2(0.707
5.3137 tan$ = lp-s/z aU "lT.s-h )an ,{T-gn
Jo.zo ,tQ=6.z6s.lT.s-h Ji an
[- r r 1.20 f1 s _
s.3137 tan$ = Q= 6.263 lJt.s- hJh dtr
L-tr-l,,lo^ Jo
5.3137 tan$ = -t l}y trigonometric substitution:
Kt.zo).r,, - (O.2o1arz1
tanf Let L : 1.5 sin2 0
= 7.30726
0 = L05.1.70 Ji =1..2242 sin}
dh=3sin0cos0d0
when ll = 0,0 = 0o
-69 whenft=1.5,0 =90=n/2
water flows through a paraboric weir that is 2 m deep and 2 m wide
at the top
;x3ffi ffl*Ti.n*d
or 150 m' Assumins c = 0.d:J;;'*";'il dischargi
pn/2
0=62rts
)o
Solution
Q= 28.782 [J# e.o,' ,a,
Jo
By Walli's formula:

o=m.rrr[ffi,+]
Q = 5.5336 m3/s (theoretical discharge)

Actual discharge = CQ = 0.65(5.5336)


Actual discharge = l.g)l sP/s
37 O F,:f,?,:I'ffiasurernent FLU'D FLUID MECHANICS CHAPTER StX 2-r l
& & HYDRAULICS Fluid Flow Measurement J, '
Problem 6 - 70
A trapezoidal weir havinq Problem 6 - 71
slope of 1H to 2V discharges 50 m3/s
under A sharp-crested suppressed rectangular weir 1 m long and a standard 90-
constant head of 2 m. Find :1d".
the leng-th of the weir ussumir,i C
= 0;;] degree V-notch weir are placed in the same weir box with the vertex of the V-
Sotution notch weir 150 mm below the crest of the rectangular weir. Determine the
head on thd rectangular weir when their discharges are equal. Use Francis
formula.

Solution
Let H be the head on the rectangular weir:
For the rectangular weir: (Hp = H)
_--L Qn ='L'84LHns/z
=1.84(1)ps/2
QR=1.8Ap/2
Consider the horizontal strip shown (treated
as an orifice under head ft)
dQ = CdA
'dA=xdlr
J4 For the V-notch weir: (Hy = H + 0.L5)
Qr = 1' H',F/z
Qr=1.4(H + 0.15)5/2
x=L+22
z=1/z(2_l) lQ^ = Qrl
x=L+2[lze-h)]=t+2_lt L8An3/2 = 1..4(H + 0.15)s/2 Square both sides
1..727 H3 = (H + 0.15)s
dA=(L+Z-h)ittt
By trial and error:
dQ=cJzs&+z-h)dhJfsh H = 0.891 m
f?
a= clzi +2h1/z -nr/rh,
Jlrnlr,
Q= cJzs
ftrrr,, +!n3/ -tr',,l:
+!p1z/z -?ef /,
:o=0.6J2s l3rpy,, f
L = 9.1.8 m

Using the combined rectangular and


triangular weir formulas:
O= ?CJzsLpt/z+ t cJz_stan$in
From the figure, tan =1
$
= t e.6) J2s
s0
L=9.18m
L(2)3/2 +
t p 6)
Jzs ( !11z1un
CHAPTER SIX
372 Fluid Flow Measurernent
MECHANICS
HYDRAULICS Fruid Frow;:*iljfffi 373
6-75
Problem 6 - Tz length of a Cipollefti weir should be constructed in order that the head
flow will be 0.96 m when the flow rale is 3.76 rnl / s?
I1:": O being discharged througtr a
container th"ti';;;;ume 150_rr
pipe directly into Ang.2.15m
of 6 m3. Find *.11;d'11"ter-
ir rhe rime required to
firr the.""",""i,1,i#:lyJrrfi*il:l'"";,n
Ans:1"63 lroblem6 - 76
Oll is flowing upward through a Venturi meter as shown. Assuming
Probrem 6-7g dlacharge coefficient of 0.984, calculate the flow of oil.
Calculate the discharge Ans:'1,58L/ s
through the submerged
orifice shown in the
figure.
Ans:109lit/

140 mmA
C = 0.75

Problem 6 - 74
The truncated cone show. hro a zn.
rq,iJ;;;:;#;;:"#*'J,;r'i;- How Iong does it take
to draw the

Ans: 30.4 minutes

Problem 6 - 77
mmo A Venturi meter having a throat diameter of 150 mm is installed in a
horizontal 3O0-mm-diameter water main, as shown. The coefficient of
discharge is 0.982. Determine the difference in level of the mercury columns
of the differential manometer attached to the Venturi meter if the discharge is
142L/s.
Ans:h = 255 mm
, CHAPTER SIX CHAPTER sEvEN
5^qI + Fluid Flow Measurement
FTUID MECHANICS
A HYDRAULICS
Fluid Flow in PiPds 775

300 mm A
Chapter 7
Fluid Flow in PiPes

DEFINITIONS
Pipes are closed conduits through which fluids or gases flows. Conduits
may
flow full or partially full. Pipes are referred to as conduits (usually circular)
which flow iull. Conduits flowing partially full are called open charlnels,
Problem 6 - 78
which will be discussed in Chapter 8.
Determine the head on a 45o v-notch weir for a discharge of 200 L/s. Use c
are two
o.57.
Fluid flow in pipes m3y be steady or unsteady. In steady flow, there
Ans:660 types of Ro* tt"rat exisi; they are called laminar fluo and turbulent floro.

Problem 6 - 79 Laminar Flow


For the sluice gate shown, if C", = 0.98, what is the flow rate? If C, = 0.62, The flow is said to be laminar when the path of individual fluid particles do
is the height of the opening?
not cross or intersect. The flow is always laminar when the Reynolds number
Ans: Q = 9.7 m3 / s; y = 1,45 m R. is less than 2,000.

Turbulent Flow
The flow is said to be turbulent when the path of individual particles are
irregular and continuously cross each other. Turbulent flow normally occurs
when the Reynolds number exceed 2,000'

Laminar flow in circular pipes can be maintained up to values of & as high as


50,000. However, in such iases this type of flow is inherently unstable, and
the least disturbance will transform it instantly into turbulent flow. On the
other hand, it is practically impossible for turbulent flow in a straight pipe to
persist at values tf & muctr below 2000, because any turbulence that is set up
will be damped out by viscous friction.

Critical VelocitY
The critical velocity in pipes is the velocity below which all turbulence are
damped out by tne viscosity of the fluid. This is represented by a Reynolds
number of 2000.
37 6 F,:ffJ,:[i:",?[, FLUID MECHANICS
& HYDRAUTICS
FLUID MECHANICS CHAPTER SEVEN
& HYDRAULICS Fluid Flow in Pipes 377
REYNOLDS NUMBER
Reynolds number, which is dimensionless, VELOCITY DISTRIBUTION IN PIPES
is the ratio of the inertia force
viscous force.
Laminar Flow
For pipes flowing full The velocity distribution for
laminar flow, at a cross section,
follows a parabolic law of
variation with zero velocity at
the walls. In circular pipes, the
velocity varies as the ordinates
of a paraboloid of revolution
with its average velocity equal
where: to one-half of its maximum
u = mean velocity velocity.
in m/s
D = pipe diameter in meter x
v = kinematic viscosity of the Figure 7 - 1: Laminar flow velocity distribution
fluid in m2/s
p = absolute or dynamic viscosify
in pa_s The equation for the velocity profile for laminar flow is given by:

For non-circular pipes, use D


= 4R , then the formula becomes; ,=o,-!J'r-,
- 4ttL Eq.z-S

Average velocity, u =7/xh Eq.7 - 6

The velocity at any distance r from the center of the pipe may also be
computed using the squared property of parabola:
x=a'' 'uf

r'
a
ro'
al x= -;-r,
ro-
u=ac-x
Table 7 - l: Viscosity and Density of Water
at 1 atm
Temp, oC
p, kg/m3 'v,
trr, Pa-s m2fS
Eqr,7'v
0
10
1!9q
1 {)0(l
r./oo x Iu- 1.788 x 10-6 "=rrr\r-*)
1.307 x 10'3 I.JUI x 104
20 998 1.003 x 10-3 1.005 x 10 6
40
996 0.799 ;TO; 0.802 x 10-6
where h1 = head losf in the pipe
qn 0.657 x 10-3 U.ObZ x 10{ L = pipe length
988 U.5.lE x 10-r
6U '0.555 x 10-6 ro = pipe radius
983 0.467 x 1o'3
70
0,475 x 10-5 ?c = centerline or maximum velocity
978 I 0.405 x 10-3 0.414 x 10{
UU 972 0.355 x 10'3 n ?A( - rn-5
p = absolute viscosity of the liquid
90 y = unit weight of the fluid
96s o316;10-3- 0.327 x 10'6
100 958 0.283 x 10r 0.295 x 10{ z = velocity at distance r from pipe center
- . 7.) = average velocity
378 F,:fi'J:[Tf#, FLUID
FLUID MECHANICS CHAPTER SEVEN
& & HYDRAULICS Fluid Flow in Pipes 379
Turbulent Flow
The velocity distribution for turbulent
flows varies with Reynolds numberi
where: r, = maximu-m shearing skess in the pipe
the walr and increases more .upiary' for /= friction factor
I::1::
rrom .":liiirr
the walls ",
as compared to laminar flow.
a short dis ? = mean velocity

SHEARING STRESS IN PIPES

Figure 7 - 2: Turbulent flow velocity distribution


1
The velocity u' at any point r in a pipe _t_
' of radius r, and center velocity z. is:

1o
p Shearing stress
distribution

Consider a mass of fluid of length L and radius r to move to the right as shown
in the figure. Due to head lost lru the pressure p2 becomes less than pr.
,= 6*L$,{7)a -2.M.f f atog-
r,- The shearing stress, t,, at the surface of the fluid can be found as follows:

[XFr, = 0]
Fr-F2-f,=g
The centerline or maximum velocity fr= P1 - F2
is given by:
T, A, = pt At' pz Az
r," x 2x x L = pt x nf - pz x nx2

DA

Combining Eq.7 - 9 and Eq. 7 _ 1L, and.solving


'2L#x-D'
rs= Eq.7 -13
for o gives the following:
380 Filf,'J:11:"#|", FLUID MECHANICS FLUID MECHANICS CHAPTER SEVEN
& HYDRAULICS & HYDRAULICS Fluid Flow in Pipes 38 I
Multiply and divide this equation by unit weighf
y:
Pt-Pz HEAD LOSSES IN PIPE FLOW
v Head. losses in pipes may be classified into two; the major ltead loss,which is
2L caused by pipe friction along skaight sections of pipe of uniform diameter and
ou, Pt:Pz = /rr (head loss), then uniform roughness, and Minor lrcad loss, which are caused by changes in
velocity or directions of flow, and are commonly expressed in teims of kinetic
energy.

MAIOR HEAD LOSS, hf


A. Darcy-Weisbach Formula (pipe-friction equation)
It is seen from this equation the shearing skess at the center of the
lhat
= 0) is zero and varies linearry pipe (x
with r. The iraximum shearing stress, ro, is at - fLa2
the pipe wall (at x = r). Eq.7
' D2g -1,8

_ _lht friction factor


'o- Er Eq.7 -ls /=
L = length of pipe in meters or feet
D = pipe diameter in meter or feet
4L a' 2g "Y" ? = me€ul or average velocity of flow in m/s of ft/ s

For non-circular pipes, use D = 4R, where R is the hydraulic radius defined in
Shear Velocity or Friction Vetocity, v" Eq.7 - 4

For circular pipes, the head. loss may be expressed as:

,. _ .fL
tu= 8Q2
Eq'7-1e
D#
0.0826fLO2
ry= ti+ (S.1. unit) Eq.7 - 20

where Q is the discharge.


CHAPTER SEVEN
382 Fluid Flow in pipes FLUID FLUID MECHANICS CHAPTERSEVEN 2O2
&HY & HYDRAULICS Fluid Flow in Pipes JlrJ
Value of f:
4. For rough pipes, where 6r < 0.3e : (K6rm6n)
For Laminar Flow:

.rra Eq.7 -27


#=zr"s(e)
where e= absolute roughness, mm
e/D = relative roughness (dimensionless)
6r = nominal thickness of viscous sublayer

5r= Eq.7 -28


For non-circular pipes, use Eq. 7-_ 22 with D ='4R.

For Turbutent Flow:


5. For smooth and rough pipes, turbulent (Colebrook equation)
1' For furburent flow in smooth and
rough pipes, universar resistance
laws can bq derived from:
Eq.7 - 29
#='u"d(4?.#)
This equation was plotted in 1944 by Moody into what is now called
the Moody chart for pipe friction.
where zr" is the shear velocity or friction velocity.
6: Haaland formula. This is an alternate formula for Eq. 7 - 29: This
varies less than2% fromBq.T - 29.
2. For smooth pipes, It" between 3,000
and 10O000: (Blasius)

=asror[ff.(*n)"']
f Eq.7-30

3. For smooth pipes with & up to about 3,00O000


CHAPTER SEVEN
384 Fluid Flow in pipes FLUID MECHANICS FLUID MECHANICS CHAPTER SEVEN
& HYDRAULICS & HYDRAULICS Fluid Flow in Pipes 385
Table 7 - 2: Values of Specific Roughness
for Common pipe Materials
B. Manning Formula
Material Roughness, r. The manning formula is one of the best-known open-channel formulas and is
ft mm. commonly used in pipes. The Formula is given by:
)teet;
et rlrl r r lstot, ilEw 0.00016 0.05
JrqI [g)), I lgw 1
vvrIIItEtLtoI.
0.000007
0.0001s
0.002 ' 'n 1: O4sgrlz (S.I. units) Eq.7 -31.
0.046
Riveted
0.01 3.0 1.486
Ir n. 0.007 2.0 =
a 'n R2/?gt/2 (English units) Eq.7 -22
vqqt, I rgyt
0.0008s 0.26
Yvr vugt tt, I tew
0.00015 0.046 where r =roughness coefficient
0.000s 0.1s
rvPr.qtrEu R = hydraulic radius
.
urq5s. L/td!vn, new
LoJt
_ 0.0004 0.L2
S = sl6pe of the energy gradeline = lry/L
0.000007 o on?
rtdsrrc: urawn [t btng
0.00000s
Smooth
0.001s It,
Concrete; - Smooth Substituting S = and R = D/4toEq.7
- t - 31 and solving for h7
ffi - 0.00013 0.04
0.007 2.0 ,=
r\suust r Jil l
0.000033
0.0016
0.01 +(z)'''(+)"'
0.5
('+\''' = 2'5129!*t oYuqrs
square both
vvrrr,qtD
sides and sorve for /y c
I i ] D2/l

- rr= 4E (s.r. units) Eq.z -zZ

o For non-circular pipes, use D = 4R


t'
o
o
6
g
co For circular pipes:
.9
o
rL
=
e
o
0
-._a _ a
--|F
d

to-

10'2"6n 10.2,10s 100


,trr= L0.29n2LO2
--==aii* (s.I. units)
J Eq.7 -34
Drolr
Figure 7 - 3: Moody Friction Factor
Chart. This chart is identical
to Eq. 7 _ 29 for turbulent flow.
The value of n is given in Table 7 - 4.
CHAPTER SEVEN CHAPTER SEVEN
^A '
5tJ9. Ftuid Ftow in pipes
FLUID MECHANICS
& HYDRAULICS Ftuid Flow in Pipes 387
C. Hazen Williams Formula Tabte 7 - 4: Values of n to be used with Manning Formula
The Hazen Williams formula is widely used in waterworks indusky. n
Nature gf surface Min Max
formula is applicable only to the flow of water in pipes larger than 50 mm
Neat cement surface 0.010 0.013
in.) and velocities less than 3 m/ s. This formula was designed for flow in -Wood=tave
piFe 0.010 0.013
pipes and open channels but is more corunonly used in pipes. 0.014
Plank flrrmes- nlaned 0.010
Vitrified sewer DiDe 0.010 0.017
English Units: Metal flumes, smooth 0.011 0.015
Concrete, precast 0.011 0.013
Cement moftar surfaces 0.011 0.015
Plank flumes. unDlaned 0.011 0.015
For circular pipes flowing full, this formula becomes Common-clav drainaqe tile 0.011 0.017
Concrete. monolithic 0.012 0.016
Brick with cement mortar 0.012 0.017
Cast iron - new 0.013 0.017
Cement rubble sudaces 0.017 0.030
S.L Units: Riveted steel 0.017 0.020
corruoated metal pioe 0.021 0.025
Canals and ditches, smooth earth 0.017 0.025
Ptetat fl umes. corruqated 0.022 0.030
Canals:
For circular pipes flowing fulf this formula becomes: 0.025 0.033
Dredoed in earth, smooth
In rock cuts, smooth 0.025 0.035
Rouoh beds and weeds on sides 0.025 0.040
Rock cuts, jagged and irregular 0.035 0.045
Natural streams
Smnothect - 0.025 0.033
R.ouohest 0.045 0.060
where: G = Hazen Williams coefficient Very weedy 0.075 0.150
D = pipe diameter in Su- Flud Mechanics by Daugherty, Franzini, & Finnemore
R = hydraulic radius
S : slope of theEGL= lU/L

Table 7 - 3: Recommended Value for C1 for Hazen Williams Formula MINOR HEAD LOSS
DesCriplion of Fipg Valuq oJ Cr Minor losses are caused by the changes in direction or velocity of flow' These
Extremelv smooth and straioht oioe 140 changes may be due to sudden contraction, sudden enlargemant, ualaes, bends,
'f the length
New. smooth cast iron oines can usually be neglected
130 und a-ny othlr pipe fittings. These losses
Averaqe cast iron oioes
Vitrified sewer oines
110
110
of the iipellne ii greater than 1500 times the pipe's diameter' _However, in
Cast iron oioes. some vears in service 100
short pipelines, because these losses may exceed the friction losses, minor
Cast iron Dipes. in bad condition BO losses must be considered.
New riveted steel 110
Smooth wooden or wood stave t20
388 Ff,fi'J:[::",?]", FLUID CHAPTER SEVEN
&r Fluid Flow in Pipes 3gg
A. Sudden Entargement
The head Ioss, m, across a sudden
enlargement of pipe diameter
is:
I
t.2

1.1
\2/A1 =4
A,
t\
1.0 --+
0.9
where: ?r = velocity before enlargement,
m/s
r,z = velocity after enlargement, m/s Y.
0.8
// .tttttt
b 0.7 H 3ad Loss = K(vr - v)2129 -
Another equation for the head It ttttll
3
determined experimentally by
ross caused by sudden enlargements
Archea u-iJ"!'.,"r, ug
E
o.o
I
0.4

0.3
I
o,2

A"special applicarion 0.1


40 and Eq,. 7 _41 is the discharge
into a reservoir. The ?f lq inT -the
water reserviir has no velocity, so a
from a p 0
0. 10" 20. 30. 40" 50. 50" 70. 80" 90. 100. 110'120" 130" 140. 150. 160. r70" 180'
head is lost. full velocify
Angle 0 between diverging sides of pipe

Figure 7 - 4: Head-loss coefficient for a pipe with diverging sides.


B. Gradual Enlargement
The head ross' m' across a graduar
conicar enlargement of pipe Table 7 - 5: Loss coefficients for sudden contraction
diameter is:
DzJDt 0.0 0.1 0.2 0.3 0.4 0.5 0.6 0.7 0.8 0.9 1.0

& 0.50 0.45 0.42 0.39 0.36 0.33 0.28 0.22 0.15 0.06 0.00

The approximate values of K


are shown in Figure Z _ 4.
A special case of sudden contraction is the entrance loss for pipes connected to
a reservoir. For this case, the values of K. are as follows:

The head loss, n1 across a sudden


contraction of a pipe is:
Projecting connection .............................& = 1.00
Rounded connection... ......K. = 0.05
Pipe projecting into reservoir................K. = 0.80
Slightly rounded entrance .....................& = 0.25
where: K" = the coefficient_of sudden
' Sharp-cornered entrance........................(. = Q.$Q
conkactiory See Table 7 _ 5
u velocify in smaller pipe
=
CHAPTER SEVEN
3eo FlJf,'J:[::",il, FLUID MECHANICS
& HYDRAUI-ICS
Fruid Flow in Pipes 391
C. Bends and Standard Fittings hL *oun."
The head loss that occurs in pipe fittings, such as valves and elbows, and
bends is given by:

The approdmate values of K are given in Tablez - 6. K values vary not only
for different sizes of fittings but with different manufacturers. For these
reasons, manufacturer's dati are the best source for loss coefficients.

The head loss due to pipe fittings mdy also be found. by increasing the pipe t,
length using the values of L/D in Table z - 6. For very smooth pip"s, 1t-is
For pipe with constant diameter, the difference between the water levels in the
better to use the K values when deterrnining the loss through fittings. see
Problem 7 - 14.
pie/ometer tubes. tf the pipe is horizontal and with uniform size, the
iiff"r"r.u in pressure head measures the head lost between the two points.
\
Table 7 - 6i Loss factors for pipe fittings total head
If the pipe is very large such that the velocity head is very small, the
Fitting K LID lost HL can be taken as equal to H.
Globe valve, fully open 10 350
Angle yalve, fully open 5 175
Close-return bend 2.2 75
T, throuqh side outlet 1.8 67 PIPE CONNECTING TWO RESERVOIRS
Short-radius elbow 0.90 32 When one or more pipes connects two resetvoirs as in the figure shown, the
Medium-radius elbow 0.75 27
total head lost in utl ttre pipes is equal to the difference in elevation of tl're
Lono-radius elbow 0.60 20
45q elbow 0.42
liquid surfaces of the reservoir.
15
Gate valve. wide ooen 0.19 7
Gate valve, half oDen 2.06 72

HL=H
--,tn\
PIPE DISCHARGING FROM A RESERVOIR
The figure shown below shows the conditions of flow in a pipe of uniform
diameter discharging from a reservoir into open air. The lr"1o.ity head and
the pressure head in the liquid surface of the reservoir are ,ero. if there will
be no head losf the velocity head could have been equal to H, which is the
distance between the water surface in the tank and the exit end of the pipe and
the velocity of flow could have been a = ,\rfr, but such is not the case due to
losses.

--J&.r
3e2 F,:fi'J:li:x:il, &HYDRA
FLUID MECHANICS CHAPTER
Fluid Flow in
SEVEN 2612
il J
Pipes
& HYDRAULICS
PIPES CONNECTED IN SERIES
For pipes of different diameters connected in series Q= Qr+ Qz+ Qt Eor.7 - 49
as shown in the figr HL=lrn=ltr-=hn Eq.7-50
are atl equal and tr.,e totuiieuJior. o
|;1""::**'^.11's-,: "llpip.":
the sum of the individual head losses. "qrrr
ln the pipe system shown, pipe 1 draws water from reservoir A and leads to
function C which divides the flow to pipes 2 and 3, which
join again in
function D and flows through pipe 4. The surn of the flow in pipes 2 and 3
!
I-- equals the flow in pipes 1 and 4. Since the drop in the .energy grade line
between C and D is equal to the difference in the levels of piezometers a and b,
then the head lost in pipe 2 is therefore equal to the head lost in pipe 3.

o/v

Qr

If the pipe length in any probrem is about 500 diameters, the


error resurting
jrom-ngglecting minor losses wil ordinarily not exceed 5%, and. if the
pipe
length is 1000 diameters or more, the effect of minor
rosses can usuaily be The necessary equations for the system are:
considered negligibre' Neglecting minor losses, trre
head lost becomes: Qr= Qt ) Eq. (1)
Qt= Qz+ Qt ) Eq. (2)
hp= hp ) Eq. (3)
HLas= lf1+ lry+ 11,0 ) Eq. (a)
If, however, it is desired to include minor losses,
a sorution may be made first
by neglecting them and then correcting ar," ,"r"ro
to correct them. Note: The number of equations needed to solve the problem must be equal to
the number of pipes.

PIPES CONNECTED IN PARALLET


3e4 ;,Xfi']:[i:"#]",
FLUID MECHANICS
& HYDRAULICS
,,:i#,lff:?Y# 3es
EQUIVALENT PIPE El. 100
If a pipe system (O) is to be replaced with an equivalent single pipe (E),
equivalent pipe must have the same discharge and head loss as the
pipe system

Qo

Head loss = Ho

A6 Qt+ Qz= Qt
Equivalent single pipe, E Qt
Head loss = Hr

Types of Reseruoir Problems


TypeL:Giventhedischargeinoneofthepipes,orgiventhepressureatthe
;,rr,.tior", P, and the requiied
is the elevation one of the reservoirs or the
diameter or length of the one of the pipes, and

In the figure shown below, the tfuee pipes 1, 2, and,3 connects the three Type2;Givenallthepipepropertiesandelevationofallreservoirs,firrd
c respectively and with all pipes meeting at a common
reservoirs A, B, and Ure flo* in each pipe, which can be solved by trial and error'
junction D.
(elevation) of
In any of these types, the main objective is to locate the position
th" u.ergy at thllunction p. This.position represents the water.surface of
an
El, 100 il . _-.i piezometer
imaglnai; ,"r"rrroi, at P. The difference in elevation between this surface and
hn the Iurface of another reservoir is the head lost in the pipe
leading to that
)'

l\\
A
reservoir (See figure aboae).
h2

\0,

)H
gplt Et. 80
B
Procedure in Solving Reseruoir Problems:
Type 1:
Qt= Q2+ Q3 1. With known flow in one pipe leading to or flowing out from a reservoir
Junction, P of known elevation, solve for its head lost /r7'
ar*: 2. Determine the elevation of the energy grade line at the
junction of the
El. 50
pipes(P)byaddingorsubtracting(dependingonthedirectionofflow)
the head lost in thI pipe from the elevation of the water surface
in the
reservoir.
3e6 F,Xf,'J:[::"j#, FLUID MECHANICS i |LUID MECHANICS CHAPTER SEVEN
& HYDRAULICS & HYDRAULICS Fluid Flow in Pipes .3gl
3. If the known value is the pressure at p, the elevation p,
elevation of p + pp/y.
of is thei 3. After determining the direction of Qz (say towards reservoir B), express
I
all the head lost in terms the other, say in terms of h7. Letl4. r.
Draw a line from p' to the surface of the other
reservoir. These rines,
represent the EGL's of each pipe. The difference
in erevation between,
l"; i. ;;;r;;.- " "="
El. 100
P'and the surface of rhe reservoir is the hu"d
5. Solve for the discharge ,

Type 2: (See Problem 7 - 65)


L. Given all elevation and pipe properties, determine
in each pipe. Of course, the hig,hest reservoir
and the lowest always have an iilflo*, but
have an inflow or outf,low.
the direction of flow
always have an outflow
the middie,"r"*"i, (B) may
I
50m

2' To find out the direction of flow in pipe 2,


assume that
P'is at elevation B, then the values oi h7 ana h1t Qz = 0 such that
c*ro"i.r""o.-ii" uT
shgw_n, lry'1= 29 lry = 96 *j. Witf.,'i,a u^a),r, t ,o*n, solve
!fu1e
and If >
1 and
lor Qr Q:. Q, es, then pz is towards B and I/ is above reservoir
B. If Qr"< Qg, then ez is away from B and p, i,
U"io* ,"*r".O, With all head lost /rJ expressed in terms of x, all flow e can also be
expressed in terms of x (usually in the form a.lx a[
).

Example, if Darcy-Weisbach or Manning formula is used, lyvaies with qz.


lhy: x Pz1
h7= x= KlQf
Qt= K\ Ji ) Eq. (1)

lry=29-x=KzQz2
gr= Y'rJfi]| ) Eq. (2)

hp=50- x= KtQ*
gr=6'r169-r )Eq.(3)
[Qr=Qz+Q:]
K', Ji = K'rJ2oi + K'31G0;;

simplify the equation and solve for r. we may also use trial-and-error
solution.

Once x is determined, substitute it value to Equations (1), (Z), and (3)


solve for Qt, Q7 and Qg, respectively.
3e8 F,:f,'J:[T[:[, FLUID
&
FLUID MECHANICS
& HYDRAULICS
CHAPTER SEVEN
Fluid Flow in Pipes 399
PIPE NETWORKS
The following conditions must be satisfied in any pipe network: q---LKQ.2
-" Eq.7-53
1. The algebraic sum of the pressure drops (heid loss) around any zZKQ,
loop must be zero and,
2. The flow entering a junction must be equar to the flow leaving it. In applying the above equation:
2KQ,2 = algebraic sum of the head'loss in the circuit (clockwise positive,
The first condition states that there can be no discontinuity in pressure'( counterclockwise negative)
pressure drop through. any route between two junctions must be the same tI(Q, = absolute sum without regard to direction of flow (clockwise
The second condition is a statement of the law of continuity . positive, counterclockwise positive)

Pipe network problems are usually solved by numerical methods us The correction a is added or subtracted from the assumed flow in order to get
computer since any analytical solution requires the use of marry simultaneo the true or corrected flow, It is added if the direction of flow is clockwise and
equations, some of which are rronlinear. subtracted if counterclockwise,

The general formula in computing the correction cr can be expressed as:


Hardy Cross Method
The procedure suggested by Hardy Cross requires that the flow in each pi
ZKQO"
be assumed so that the principle of continuity is satisfied at each junction. cx, = ---------------- Eq.7 - 54
correction to the assumed flow is computed successively for each pipe loop nLKQo"-l
the network until the. correction is reduced to an acceptable value.
Where n = 2 for Darcy-Weisbach and Manning formulas and n = 1.85 for
Let Qo = ilssumed flow Hazen- Williams formula, The value of K are as follows:
Q true flow
=
cr = correction then; o'oY:fL
Darcy,K= Eq.7 -55
Q= Q,+ o
10.69tt2L
Using Darcy.Weisbach formula:
MElnnrng, K
-.-_*-_-.r, --= Eq.7 -.56
D1,6/3
0.0826[LQ2
Itt_ Hazen-Williams,K= Eq.7 -57
'pt - ffi
Ll U
lry= Y9'
IKQ2 = 0
EK(Q,,+61)2=0
2K Q,2 + 22Kc,.Q,+ IKcr2Q, = 0

If c is small, the term containing o2 may be neglected.


Hence;
ZKQoz + ZZKq,Qo= 0
4oo F,:f,|J,:Ii:"j#, FLUID MECHANICS
& HYDRAULICS
CHAPTER SEVEI!
rr"iJ iri*ji pip"' 40 I
Problem 7 - 4
Oil having specific gravity of 0.869 and d.ynamic viscosity of 0.0814 Pa-s flows
Problem 7 - I, through a east iron pipe at a velocity of 1 m/s. The pipe is 50 m long and 150
mrn in diameter. (a) Find the head lost due to friction, and (b) the shearing
Water having kinematic viscosity v = 1.3 x 704 m2/s flows in a 100-mm..
sLress at the walls of the pipe.
diameter pipe at a velocity of 4.5 m/ s. ls the flow laminar or turbulent?
Solution
Solution
vDP
(a) 4=
n - oD
r\a -
v
4.5(0.1)
p
- 1.3 x 10-6
(1)(0.1sX1000 x 0.85e)
R,=346,154 > 2000 (turbulentflow) &= 0.0814
Ru = 1,601 < 2,000(laminar)
Problem 7 - 2
oil -64
t=
of specific gravity 0.80 flows in a 200 mm diameter pipe. Find the critical ,R"
velocity. Use p = 8.14 x 10-2 Pa-s. -
Solution
f= 64
' 1.601.
=o.o+
,
At critical velocity in pipei,
aDP
R. = 2000
tr,=
' DZg
LYl
R,=
l.r
_ 0.04(50) (1)2
a. (0.2)(1000 x 0.80)
2000 = 0.15 2(e.81)
8.14 x 10-2 lrt= O.0e m
tt, ='1.0175 m/s

(b) _ yhrD
_,O--T

(9810 x 0,869)(0:68)(0.1s)
For laminar flow conditions, what size of pipe will deliver 6 liters per second
of oil having kinematic viscosity of 6.1 x 1[t yp/ s? 4(50)
ro= 4.34Pa
Solution
For laminar flow conditions, R" < 2000.
Problem 7 - 5
QD
aD Determine the (a) shear skess at the walls of a 300-mm-diameter pipe when
&- =A water flowing causes a head lost of 5 m in 90-m pipe length, (b) the shear
0.006 _ velocity, and the (c) shear stess at 50 mm from the centerline of the pipe.

2000 =
-I)
5.1,x'l.0-6
D=0.626m=526mm
4oz F,XfiT,:1T,',I", FLUID
&
FLUID MECHANICS
& HYDRAULICS
CHAPTER SEVEN
t,iiJl"*'"-oii"' 4O3
Solution
0.1273(0.1)
(a) Shear stress at walls: (n) ^
rQ=
-------------;
1.08 x 10-"
yhlD
To= & = 118 (laminar)
4L
_ e810(sx0.3)
(b) ^ 0.1,273(0.1)
4(e0) "'= T311il5
ro= 40.9 Pa & = 843 (laminar)

(b) Shear velocity


(c) - _ 0.1273(0.7)
''"- T.o{ffi
" lb-
7,.=
1' p & = 31,351 (turbulent)

=@_
1 rooo (d\ --
l("=-
0.1273(0.1)
1.02 x 10-b
tt,= 0.2m/s
R" = 12483 (turbulent)
(c) Shear stress 50 mm from pipe center 0.1273(0.1)
vh, (e) ^ = ----------------
l<v
1.15 x L0-'
2L
Il" = 710,71,6 (turbulent)
9810(5)
= ro.osr
2(e0)
t a - _-
r\
0.1273(0.1)
= 1.3.5 Pa fi8;10=
& = 10.8 (laminar)

A fluid flows at 0.001 m3/s through a 1O0-mm-diameter pipe. Determine


whether the flow.is laminar or turbulent if the fluid is (a) lrydrogen (v 1.0g x
= Water flow at the rate of 200 lit/sec through 120-m horizontal pipe having a
1,04 m2/ s), (b) air (v = 1-11 x 1.0-s mz/ s), (c) gasoline
1v = 4.b6 diameter of 300 mm. If the pressure difference between the end Points is 280
water (v = L.02 x 1'0-6 m2/ s), (e) mercury (v = L.15 x 1.0q mz/ s), "10, ^r1ry, 1d1
or (fl grycerin'(v mmHg, determine the friction factor.
= L.1g x 1g-o pz/s)
Solution
Solution
. o.o826fLQ2
,Lt--
R"='D 'Dr
v
u- a
-._ 0.001
For a horizontal pipe, the head lost between the points is equal to
-A
tQ.1)2' the difference in pressure head, See page 376.
a = 0.1273m/s
CHAPTER SEVEN
404 Ftuid Flow in pipes FLUID MECHANICS FTUID MECHANICS CHAPTER SEVEN
& HYDRAULTCS & HYDRAULICS Fluid Flow in Pipes 405
h,= ltllz = 0.28 mm Hg (13.6) = 3.808 m of water 2
v 1 ntr _ 0.0825(0.0356)(150)Q
(0.02)s
0.0826f (120)(0.42
3.808 = Q = 0.00111 m3/s
(0.3)s
Q = 1..1'L Litlsec.
f = 0.0233

The head lost in 50 m of L2-cm-diameter pipe is known to be 5 m when a


A fluid having v = 4 x 1g's sz/s flows in a7s0 liquid of sp. gr. 0.9 flows at 0.05 m/s. Find the shear stress at the walls of the
mrong pipe having a diameter
of 20 rnm' Determine the head rost required
to maintin a ,etocity or s m7s. Pipe.
Solution Solution.
oD
l!!.LD --
"- -T
t<" = -: (e810x0.e)(6)(0.12)
v
rco)
^=
re
3(0.02)
----:---; = 1500 < 2000 (laminar) r, = 3'1,.78 Pa
4x 10-'
.64 64
";= -R,. 1,500
= 0.042667 what conrmercial size of new cast iron pipe shall be used to carry 4,490 gpm
with a lost of head of 10.56 feet per mile? Assume,f = 0.0f 9.
0.042667(750) G)2
n,=
' Lvz
D 2s o.o2 -re.81) Solution
lq= 733.95 m
,- _ 0.0826fLQ2
,l'__

'D5
Problem 7 - 9
Fluid flows thr
Q=4,4e0#"%F,hH
20-mm-diame-te1pipe, 150 m long at a
*:-".t-1," Reynolds number Q = 284 lit/sec = 0.284 rfi / s
of 1.,750. Calculate the discharge if tf," f,"ua filr?r. f*if,
L=
..
I nule t
5280 ft L m
Solution ,rilt 't2g ft
0.0826fLQ2 L ='1,609.76 m
hf=
Ds
Since& =L,750<2,000, the flow is laminar.
Itf =10.55 ft ><
5ffi
ly=3.22m
.64
"r= _R" =_
64
7750 3.22=
f = 0.0366'
D=0.576m=576mm
406 F,XfiT:[::,.#, FLUID FTUID MECHANICS CHAPTER SEVEN
& & HYDRAULICS Fluid Flow in Pipes +Ol
Problem 7 - L2 (CE Board 1988)
There is a leak in a horizontal 3,0_mm-diameter Problem 7 - L3 (CE May 2003)
pipeli.". Uprt uu* frorn Water flows from a tank through L60 feet of 4 inches diameter pipe and then
400 showed u ain"ruru of 140 kpa. no*r,ro"url'ri
f1lY""jir1 two gages lgnart
600 m apart rrto*"a u-aiirurence distharges into air as shown in Figure 20. The flow of water in the pipe is 12
lh:t^t:'tk of 125 kpa. Assu cfs. Assume n = 0.013 and neglect minor losses. Determine the following:
0.025, how much water is being lost from ,f.,"
pip". (a) The velocity of flow in the pipe in fos,
Solution (b) The total head lost in the pipe in fee!
(c) The pressure at the top of the tank in psi.

Qz
Lz = 600 r----{

0.082![LQz
'-
0.0826(g.02gx600)
D5 -
hf
(o,qf- e2
fu= 509.875Q2
Since the pipe is uniform and horizontal,
the head lost between any to
is equal to the pressure head difference.

p1:yP2
u,fi = |

soo.8;6gr,=H Solution
9.81,
Qr = 0.167 m3/s Q=12f#f s= 0.9401m3/s D=4" El. 100'
D = 4" = 0.333 ft = 101.6 mm
n = 0.073
[ho= Pz - P+ L = 1,60 feet = 48.78 m
,
v
126
509.876Q22=;;i
Qz= 0.159 rfi/s
[Q,= Q, - qr1
Q, = 0.167 - 0.159 = 0.008 m3/s
Q, = 8 Lit/sec
CHAPTER SEVEN
408 Fluid Flow in pipes
FLUID TLUID MECHANICS CHAPTER SEVEN
+o9
& & HYDRAULICS Fluid Flow in Pipes
(a) Velocity of flow in the pipe:
Problem 7 - 14
,=Q A 600-mm diarneter pipe connects two reservoir whose difference in water
A
lurface elevation is 48 m. The pipe is 3500 m long and has the following pipe
=12 flttings: 2 globe valves, 4 short radius elbows, 2 long radius elbows, and one
f (0.$3), gate valve half open. The values of loss factors for pipe fittings in given in
o = 137.785 ft/s
TableT - 6.

(b) Head lost in the pipe:


Using the equivalent length method, estimate the flow through the pipe in
HL _ 1o.z9n2 Le2 l,/s. Assunr,e/= 0.015. ,
b16/3
10.2e (0.08)2 (45.7 S) (O.s 4O\2 Solution
The total head lost in the system is equal to the difference in elevation of
HL = 1942.23 m = 6370.8G feet . the surfaces = 48 m

(c) Pressure in the tank: HL _ o.os26{LQ2


Energy equation between A and C D5
Ea-HL=Ea L = 3500 + 2[350(0.6)] + 4[32(0.5)] +.2120(0.6)l + 1"172(0.6)l = 4fr64 rn

{.29 ?y *ro-HL=
^ {.zg'y !s- +7, HL=
0.0s26(0.01sx 4o5qQ2
=48
o+L
y +rc-6;1z0.86-
O97:78-q2 +o+1oo Q = 0.86t 6r/s = 861,lJs
2(32.2)
" P
v
- 64sl.6sfeet of water
p = 421,,552.9 psf
Problem 7 - L5 (CE May 2002)
p = 2,927.45 psi In the syringe of the figure shown, the drug had p = 900 kg/m3 and p = g.gg2
Pa-s. The flow through the needle is 0.4 mL/s. Neglect head loss in the larger
Using the English units for Manning,s Formula: cylinder.
1'49
71 = R2/3 St/2 dz=10mm
n
a = '132.795 ft/ s
dr = 0,25 mm
R= D/4
R=0.332/4=0.0833ft
S= HL/L= HL/1.60
lsz .z8s = ::*(0.0s33)23(H L/ tosltrz
0.013 '
HL = 5355 feet

(a) Determine the velocity atpoint B in m/s.


(b) What the Reynolds number for the flow in the needle.
(c) Determine the steady force F required to produce the given flow.
4to F,:f,';:iT#I", FLUID MECHANICS
& HYDRAULICS
CHAPTER SEVEN
n"iJ ir"*ii pipii 4l I
Solution
Solution
(a) Velocity at point B:
Colebrook Formula:
" ,r= I - o'4x10{
A
ua = B,14BZ
+(0.0002s)2
$/s #=,*(##)
(D) Reynolds Number:
R" =
aDo
---- #=,^r(Tf.#fu1
p
_ 8. 1487 (0j 0_0_q2s

0.002
xe00)
#=,^r(ryf.#fu)
& = 9'1.6.73 (laminar flow)
Solve for/by trial and error:
(c) Force F:
Energy Equation between A and Bl
/= 0.0s1s

Ea-HL=Es
Using the Moody Diagram,/= 0.05
",2 ..
uA *PA )
+zA-HL=+:*pB
29 y ' 2g'y
+zB

r.'a = 0 (negligible)
HL = bin the needle
0.05
0.04
Since the flow is laminar:
0.03
f = 54/R"- 64/91.5.73 0.02
f = o.06gs 0.015
U
0.01 'b
HL= 0.008 3
o 0.006 I
o t
o 0.004 o
HL = 18.89 m 5
o
o
o 0.002 o
o+ Pe +o-18.89 - @J'4s7)z +o+o
'tr
r ':
y 2(e.87)
0.001
o.ooo8
.!!
0.0006 E
I
Pa = 196,68'l Pa 0.0004

0.0002
Force, F = pA x Areaof piston
=196,681. x f (0.01),
Force,F=15.45N
td z,'to' 105 2,10t .to! 2,10. 1ot 2*1ol

Determine the friction-factor for flow having a


Reynords number of 5,000 r
relative roughness (e/ d) of 0.075 (transition zone)
using Corebrook formura.
4l. t G
) CHAPTER SEVEN
FLUID MECHANTCS FLUID MECHANICS
Fluid Flow in pipes
& HYDRAULICS & HYDRAULICS
_GHAPTER.SEVEN
Fluid Flow in Pipes 413
Problem 7 - L7
The velocities of flow in a r.-m-diameter pipe are 5 m/s
or, using Eq.7 -?
on the centerrine and
4.85 m/s at r = 100 mm. Determine discharge ifl= . '0 = 7)c - 3.zs
^lr' =5 - 3.75(0.2692)
1l P
Solution n=399mfs
- The velocity at any point is given by Eq.7 - g:
Discharge, Q= Att= t (1)'?(3.99)
u = t)c -5.75,1:!-bg-Js-
VP rn-r Discharge,Q=3.13m7s

Pipe radius, ro = 500 mrn


Centerline vglocity, (,. = 5 m/s
Velocity at r = 100 rulL u = 4.85 m/s oil or sp' gr' 0'9 and dynamic viscosity p = 0'04 Pa-s flows at the rate of 60
liters per second througtr 50 rn of 120-mm-diameter pipe. If the head lost is
4.85=5 -s.7s
t;: . soo
l- 6m, deterntine (a) the mean velocity of flow, (b) the type of flow, (c) the friction
I p --. 5oo _ 1oo factor f, (d) the velocity at the cenierline of the pipe, (e) the shear stress at the
wall of the pipe, and. (fl the velocity 50 mm from the centerline of the pipe'
= 0.2692
Solution
(a) Mean velocity
tr _w. o 0.05
i;-Va t
A iq::)z
fn' o = 5.3'1, m/s
= 0.2692
i ,- (b) Type of flow
fo2 = 0.5792 aDp 5.31(0.12)t1000(0.9)l
o - p -
._
t-+
0.5797
0.04
o'
R, = L4,337 > 2000 (turbulent flow)
FromEq.T-11:
/_\ (c) Friction factor
u.=?(1+t.sz,tf )
,lt,=-o.o826fLQ2
s=r[r*r.sa 'D5
,_ o.o826f (50x0.06)2
-
s =, (t+ 1.0129) (0.12)s
\u) ,f= 0.01004
5=o+1'.0126
o = 3.99 m/s (d) Centerline velocity
o,= o(1. + 1.33f /2)
= 5.31[1 + 1.33(0.01004)1/'z]
a,= 6.02m/s
4t+ Ff,f,';:}i:"#il, FLUID MECHANIC$
&
FLUID MECHANICS
& HYDRAULICS
CHAPTER SEVEN
Fluid Flow in Pipes 415
(e\ Shear shess at the wall of the pipe
From Eq .7 -9:
t;-
lu=a,-3.75^l:! 1

ittr =i"
YP
lro' squaring both sides: a=4.6-3.75(0.2582)
a =3.63m/s
to= tu'
p 8
Discharge, Q = Aa

. xo _ 0.01004(5.31)2 t Q.75)2(3.63)
Discharge, Q=1.6m3/s
1000 x 0.9 8
to = 31.85 Pa

UsingEq.7-16: Problem 7 - 20
What is the hydraulic radius of rectangular air duct 200 mm by 350 mm?
_ yhrD _
--"- (9810x0.9X6)(0.12) a

4L - 4(so)- Solution
. xs = 31.78 Pa
)
Hydraulic radius, R= A/P
(/) Velocity at 50 mrn from the centerline D _ 200x350
200x2+350x2
il=uc-s.7s,11" bg _!t_ R = 63.5 mm
\l P ro-t

Problem 7 - 2L
u = 5.778 m/s 100 oC flows in a 20-mm-diameter tube. what is the
Air at 1450 kPa abs and
maximum laminar flow rate? Use R = 287 J/kg:l(, lL= 2.17 x 105 Pa-s.

Solution
The velogities in a 7S0-mm-diameter pipe are measured For laminar flow, & < 2000
as 4.6 m/s and 4.4
m/s at r = 0 and r = 100 mm, respectireiy. For turbulent flow, determine o -vDP
r\'-
the p
flow rate.
-
Solution Solve for p:
p
il = u, _ s.z5 bg_r,_ RT
^l:!_
1, P to-r
. 14s0(1000)
287(100 + 273)
p b,
4.4=4.b-s.7s l:!_ --o 375
1, 375_1,00 p = 13.54 kg/mg

to u(0.02)(13.s4)
p
= 0.2582 O_ =2000
2.17 x1.0-'
v = 0.1,603m/s
CHAPTER SEVEN
416 Fluid Flow in pipes FLUID MECHANICS CHAPTER SEVEN
& HYDRAULICS Fluid Flow in Pipes 417
Q= Au
Problem 7 - 23
= f (0.0212x0.1603
A liquid having a sp. gr. of 0.788 flows at 3.2 m/ s through a lO0-mm-diameter
= 0.0000503 m3/s pipeline (f = 0.0158). (a) Determine the head loss per kilometer of pipe and (b)
Q 0.0503 lit/sec.
= the wall shear shess.

Solution
(a) Head loss per kilometer (t = 1000 m)
Glyceri^ (sp' gr' = r"26.and p 7.49pa-s)
300.mm by 450 mm at
= flows throughr qa teL(angul
the rate of roo
rectangurar conduit
fi-fec h,= t-
' LD2g
(a) Is the flow laminar or
turbulent?
(&) Determine the head lostper
kilometer length of pipe.
Solution
0.1 2(e.81)
' hr= 82'5 m
(a) For non-circular conduits;
43RP (b) Wall skess:
R, =
p r^u - )faz )SeeProblemT-1,8(e)
-. - O=
oE;
0.16
o.as
= 1.185 m/s t- 8

4p - o'3oxo'45
t o- _ 0.0158(3.2)2
R= 1000 x 0.788 8
4m.045) t, = 1.5.94 Pa
R=0.09m
or:
* = a(1.185X0.09X1000 x 1.26)
lhrD (9810 x 0.788)x 82.5 x 9.1
&=360.25<zoffiminr.) - = 4L -
1.
4x1000
ro = 15.94 Pa
(&) For laminar flow:
,- 64 64
' R. 360.75
f = 0.1774 oil with sp. gr. 0.95 flows at 200 lit/sec through a 500 m of 20O-mm-diameter
pipe (/= 0.0225). Determine (a) the head loss and (b) the pressure drop if the
,
llf=!--
fLV2 pipe slopes down at 10" in the direction of flow.
D2g
Solution
? = 4R (for non_circular pipes)
(a) Head loss
D = 4(0.09) = 0.35 m

'DL
a2 _ o.o826fLQ2
lrr=
7, =
0,1774(1ooo) (1.185)2
0.36 2e.s7)
4 Ds

lry= 35.27 m . 0.0826(0.0225)(500)(0.2)2


h,= ' ') " ' =775.2rll
' (0.2)s
4t8 F,Xf,'J:[::o'#, FLUID MECHANICS
& HYDRAULICS
CHAPTERSEVEN t
ilff[#;;fi..: 4te
(b) Pressure drop:
az2
0+0+h-ht= +0+0
2g

h= -;' +ht )
,r^2
Eq. (1)
a zg
Energy equation between L and 2 (datum
at 2): , fLrz
Er-HL=Ez
)
'U= DE
*1 * lt + 21 -HL= azz + lz *,^ D=4R=4L
zgy2gy'z P

o!' A= ip.1)2- t(0.05),


Since ar = r,r, ";'1-
29 und 29 cancels out
A = 0.005026 m2
P = nDo + rDi = r(0.1) + n(0.06)
lL + 86.82 - 116.2 - P2 +0 P = 0.50265 m
Ty
Pr - Pz P=40'oo5o25 =o.o4m
=.29.Bgmof oil 0.50265
yy 10
p1 - p2 = 29.38(9.8L x 0.95) -._u _ 1000

Pt - Pz= 273.87kPa
A 0.005026
a=1,.9896mfs=tf2
0.0232(50) 1.98962
,,r_ =5.85m
7- ' 0.04 2(e.81)
water flows commercial steel annurus 30 m rong as
-through
firyr1. Neglecting shown in the In Eq. (1):
. minor rosses, estimite the reservoir rever
maintain a flow of 10lit/sec. Assume/= O.O2g2. /r needed to
1'9895' *s.gs
tr=
2(e.81)
h = 5.052 m

60mmq

H
ll;At\t1
t:I
Problem 7 - 26
Find the approximate flow rate at which water will flow in a conduit shaped
r0A ]4 in the form of an equilateral triangle if the head lost is 5 rn per kilometer
lrJ/l
F----{ length. The cross-sectional area of the duct is 0.075 m2. Assume,F= 0.0155.
Solution 700fim o
Energy equation between 1 and 2 (datum
Solution
at 2)
Er'lV= Ez - fLa2
' D2g
. P7 +z.t-rr,='r'
{zgy29y' + lz *,^ h7=5m
L=1000m
CHAPTER SEVEN
420 Fluid Flow in pipes FLUID MECHANICSi
FIUID MECHANICS CHAPTER SEVEN
& HYDRAUI,ICS & HYDRAT'LICS Fluid Flow in Pipes 421
D=4R
R= A/P = A/(Zx) *
l$= 1.878(0.15)
-:-::# =682<2000
A = Tz x2 sin 60o = 0.075 laminar, OK
0.000413
x = 0.4'l,Gm
R = 0.025/ [3(0.41,6)] IQ-- Aol
R=0.06m Q= ito.ts1,(1.878)
D = 4(0.06) Q = 0.0332 m3/s
D =0.24m

s _ 0.01s5(1000) a2
0.24 2(e.81)
- 28 (CE
a = 1.232m/s In the figure shown, the 50-m pipe is 60 mm in diameter. The fluid flowing
lras mass density of 920 kg/ms and d.ynamic viscosity of 0.29 pa-s. The
IQ = Aal pressure in the enclosedtank is 200 kpa gage. Determine the following:
Q = 0.07s (7.232) = 0.0924 mt / s (a) The amount and direction of flow?
Q = 92.4lit/sec (b) The velocity of flow in the pipe?
(c) The Reynolds Number of the flow?
I

tlli

Heavy fuer o, flows,from A to B trrrough a


1000-m horizontal 150-mm-
diameter steel pipe. The pressure
at A is rloso tpu a'd at B is
kinematic viscositv is o.Oob+rs *r7r 35 kpa. The i
u"Jlrru specific gravity is 0.92. what
the flow rate? is

Solution

For uniform horizontal pipe, ly= P n - Pe


v
,
,Ir= 1050-35
' =112.46m
9.9'1,x0.92
Solution
Assuming laminar flow:
. -
32vLo
' gD'
32(o.oooa13)(1ooo)u
1,1 2.46 =
9.81(0.1s)2
a='1,.878m

Check:
iD
&-
+22 F,Xfl';:Ii:",',I",
FLUID MECHANTCS
& HYDRAULICS
FLUID MECHANICS
& HYDRAULICS
CHAPTER SEVEN
.,iffiliili?-p1l 423
Taking level 2 as the daturn: az = 0,471 ) Eq. (1)
Energy Er=12m f

Energy, E2: Q + l- = !- =
200,000
=22.16m Try f = 0.03:
yp8 920x9.81 InEq. (1): 0.03o2 = 0.471; a =3.962m/s
3'952(0'3)
Since Ez > E1, the flow is frorn 2 to 1. R,,- =5e,430
0.00002
Energy equation befween 2 and 1:
=+arc,[#. (#),,,)
E2- lry=
22.16
E1
- l,r= tZ
fr
ly= 1.0.1.6 m

Assuminglaminar flow (Rr < 2000) i=^o{#h.(Tr)*]


= 0.0206
128PL? 128(o.2ex5o)Q f
tt,= I
np1Dn '
10.16 - n(920)(9.81)(0.06)a
NewT= 0.0206
a = 0.00201 m3/s In Eq. (1): 0.0206 az = 0.471.; a = 4.78
4'78(0'3)
Velocity of flow, r= i!0201 &=' =7"L7oo
= 0.771 m/s 0.00002
+ (0.06)'

Reynolds Number, = 3qq = 0711(9'.99X920) =.18b8wh.(#)*]


7
RE

Reynolds Nurnber, R6 = = 135.4 < 2000 (laminar flow, OK)


T .01ee (oK),
f=o
Since our assumption is correct, then
Discharge, Q = 0.0020t m3/s x 3600 s/hr In Eq. (1): 0.0199a2 = 0.47'1.
Discharge, Q = 7.24 m/hr (from 2 to 1l ?) = 4.865 m/ s
Velocity of flow, a = 0.777 m/s
Reynolds Nurnber = 135.4 Q=Aa = f (0.3)14.86s)
Q = O.344m3/s

Problem 7 - 29
Oil, with p = 950 kg/nt3 and v = 0.00002 m2/s, flows through a 300-mm- Problem 7 - 30
diameter pipe that is 100 m long with a head loss of 8 m. €/D = 0.0002. Two tanks of a solvent (v = 0.0000613 mzf s, y = 8 kN/m3) are connected by 350
Calculate the flow rate. m of commercial steel pipe (roughness, E = 0.000046 m). What size must the
pipe be to convey 60Lls, if the surface of one tank if 5 m higher than the
Solution other. Neglect minor losses.

Ilrr=
It,,, Solution
D ,st lY=s^
8- /(100) uZ
0.3 2(e.81)
42+ F,:f,T,:[i:",',[ FLUID MECHANICS CHAPTER SEVEN
& HYDRAULICS Fluid Flow in Pipes 425
_ 0.0826/(3s0X0.06)2 D = 0.461(0.03741trs = 0.24 m
s
D5 II= 5792
0.461fl/s e/D = 0.000192
O= ) Eq. (1)
= 0.0374
Q f
,..,

, -
r\a-
aD
- tD'" _4Q The procedure has converged to the correct diameter of 240 mm.
VV - rcDv

-_
D 4(0.06) 1.246 An approximate formula for D is given as follows:
"" ;D(016664, D
) Eq. (2)
I r ^\4.25 r ,5.2 lo.o4
e-
DD
0.000046
) Eq. (3)
o =ooole12'1ff) .,n,,[#]
]
Tryl= 6.93
In EQ. (1): D = 0.451(0.03)t rs = 9.229,
= o*[,o oooo+or, "[3s{9991) *u.rr"ro-sro.oel"n(ffi)"]"-
In Eq. (2): o,=
ffi
e = 0.000046
=5450 D=0.234m=234mm
In Eq. (3): -O.nSe =0.000201'
O
Solve for new/fr.om Eq.7 - 30: what size of new cast iron pipe (e = 0.00026) is needed to h.ansport 550 L/s of
water for 1 km with head loss of 3 m? Use v = ,.0, * 1g-z rcp/ s.

#=-'u"'iff.(#)"'] Solution
o oeeim)l
il o.0g26lLQ2
= -r r,"r[s?u?. ( ,_,_
,Ll - -'----="'-=-
# ]
'D2
f=0.03592 )New/ "
_ 0.0826f (1000)(0.5s)2
D"
In Eq. (1): D = 0.461(0.0369}yvs= 0.2383 D = 1.528f /s ) Eq. (1)
In Eq. (2): D_4Q
' rDv _
4(0.s5)
o" =
ffi = S22e
nD(9.02x t0-7 )
In Eq. (3): =
,*---E-
; THq = 0.0001e3 o_ 776,366
) Eq. (2)
Solve for new /from Eq. 7 - B0: e _ 0.00026
DD ) Eq. (3)

f = -r a r" rl*. (, :3i,u)"' ] Tryl= 0.03


f = 0.0374 Lefls use this/ D = 1.528(0.03)t/s = 9.757u
11, = 776,366/ (0.7578) ='1.,024,500
e / D = 0.00026 / 0.7578 = 0.000343
426 F,Xf,?,:li:",',I",
FLUID MECHANICS
&
FLUID MECHANICS
& HYERAULICS
CHAPTER SEVEN
rrijiir"'.i"i,p"' 427
0.915 fi..22\
+(0099!43)] '1] = o'261 m
i "n.*[-ffi*
= R=
40.s1il12a-
f = 0.0't6
-" _ e810(4.6)
2(e1..s)
rc.261\
New/= 9.916 ts = 64.36 Pa
D = 1.528(0.0161t ts = g.uut
R, = 77 6,366 / (0.668) = 1,162,225
e/ D = 0.00026 / 0.665 = 0.000389 Problem 7 - 33
o oeelSe Oil with p = 900 kg/*, and v = 0.00001m2/s m flows at 0.2rfi/s through
=, n 200-
# "rl#,.r+ (
)"1 ] mm-diameter cast iron pipe 600 m long. Determine the head loss.

f = o.uae (oK)
Solution
D = 1.528(0.01637rrs = 0.671 m From Table 7 -1,,e= 0.26 mm
D = 57'l mm e/D =0.26/2oo
e/D = 0.0013
Using the approximate formula:
u
D=o6oi,,*l!Q_ I *uoool-!-]"1"'
< w= *,
I lsi,,,1 1s,,, 1 l ?- 0.2
a =6.37m/s
l, roooro'ss)''\o''u
+(o'2)'
+eo2xto-71oss;e4[uffi)"]'-
= 0.65i 0.00026'
L "I,. ,ffi?t,l K=+l
D = 0.583 m
D _ 6.37(0.2)
rv- _.nn
-:-:- 0.00001
^nn
-127,400
Problem 7 - 32
From the moody d.iagram:
water is flowing through a 915 mm x 1220 *m rectangular conduit oi length ,z
91.5 rn and a head loss of 4.6 m. what is the shear stress between the water f 0.022s
and the pipe wall?
By Haaland Formula: j

Solution
For non-circular pipes,
v lt,
#=.1,bsl;ih.(#)"']
La - f = a.0226
2L
-l\

A
R=7
428 F,Xfi';:li:Y;S, II.UID MECHANICS CHAPTER SEVEN
& HYDRAULICS Fluid Flow in Pipes 429
From the Moody diagram,/= 0.02
UsingEq.T-30
0.04

=-raro,[f.(#-)"')
0.03

0.02 f
0.0'15
6.s
o
0
6 0.025
E o.ozzs
0.01
0.008

f
,tr
I
= 0.01e7
=
'81ffi.(.#]
-r.sr"^[
l
1o'oooal'r1r

ri o.oz 0.002

0.001
0.0008
0.0006
,
-. fLo2-
ll,,- "
0.0004
D -l2s'
o.o0o2

,- _ 0.02(80) 1.832
0.00005 ' 0.15 2(9.81)
0.00001
lq=1"82m
lO3 2 . 1or 10' 2,10. .iO5
2, los
103
Reynolds number. R"
Pressure drop for horizontal pipe, Lp = p g ht
y 4=
,,.
'= L o2 _ o.ozzsloool 6322 = ee8(e.81)(1.82)
D 2S 02 4rs1) Pressure drop for horizontal pipe, Lp = 12818.5 pa
lq='139.6 m

what size of pipe is required to carry 450 liters per second of water with a
Compute the head and pressure drop in g0 m of horizontal
los.s. head loss of 3.4 m for 5000 m length? Assume friction factor y = g.g24.
150_mm-
cast-iron pipe carrvi.g iater ut 20"c *l;h;";#"Til:Til
#1',1:'ili;,halted Solution
o.0826JLd
Sotution U=
From Table 7 - L, v= 1.005 x l0-6 pa_s
- D5--
From Table 7 - 2, x = 0.12 mnr 2 tr _ 0.0826(0.024X5000)(0.4s)2
D5
D=0.895m=895mm
n,.= ,o _ 1.83(0.15)
v 1:005 * 10-7
&.= 273,"134
Pi
e 0."12
:- -'^-
D = 150 = 0.0008 water flows in a 300 nun x 400 mm rectangular conduit at the rate of 150'
l

lit/sec. Assuming/= 0.025, find the head loss per km length.


4?n CHAPTER SEVEN
I rru, Fluid Flow in pipes
FLUID MECHAN'CS
& HYDRAULICS]
FLUID MECHANICS
& HYDRAULICS ilil}I#i;;;il
CHAPTER SEVEN
43t
Solution
Problem 7 - 3A

uu=ffit A 2.5-m-diameter pipe of length 2,500 m conveys water between two


reservoirs at the rate of 8.5 m3/s. What must be the difference in water-surface
A elevations between the two reservoirs? Neglect minor losses and assume 7F =
D=4R=4(F) 0.018.

o=+
o
frffi 0.15
=0.343m Solution
For two. reservoirs, the difference in elevation between the surfaces is
" A
rr=- =
(0.3)(0.4) equal to the total head.
a = 7.25 m/s o.os26fLt
HL=hj =
,- _ 0.025(1000) 1.252 D5
' 0.343 2(9.81) 0.0826(0.018) (2500X8.s)
2

/r7= 5.8 m (2.s)s


HL= 2.75 m (difference in elevation)

7-
A 20-mm-diameter conrmercial steer pipe, 30 m long is used to Problem 7 - 39
drain an oil
tank. Determine the discharge *hen ine oil rever in ihe tant< i, g;uboru Water at 20 oC is to be pumped through 3 km of 20O-mm-diameter wreught
the
exit of the pipe. Neglect rriinor losses and assume/= 0.12. iron pipe at the rate of 0.06 m3/s. Compute the'head loss and power required
to maintain the flow. Use v = 1.02 x 10-6 m2/s and roughness e = 0.000045 m.
Solution
Solution
, fLoz
' D2g
ks{,.,=0.,,
Solve forf
Rr='D
Energy equation between O and G):
Er - hf= Ez
ut2 Pt +Z"t-lk=---:-+
or2
_Pz +22
zgy29
=-f v
0'0826(0'12X30)Q2
0+0+3 - - ; 802
- , +0+0 t 0.000046
= 0.00023
(0.02)s n'g(0.02)a -=
D 0.2
Q= 0.000179 mt/g
Q= 0.17els
4?7
I rtL
CHAPTER SEVEN
FLUID MECHANICS
Fluid Flow in Pipes CHAPTER SEVEN
& HYDRAULTCS Fluid Flow in Pipes 433
From the moody.diagram:/= 0.01f
Problem 7 - 40
A pump draws 20 lit/sec of water from reservoir A to reservoir B as shown.
Assuming f = 0.02 for all pipes, compute the horsepower derivered by the
pump and the pressures at points L and 2.

Solution

10' 2 1o1 to, 2, toi


4 '
" 10"

UsingEq.T-30
2OO mm - 5OO m

# =-,,,.-fff .(Ii)"'l Qt = Qz = 0.02 m3/s


h _ 0.0825(0.0?)900x0.02)2 = 1.033 m
i='"^r1ffi.("uT")"'] .llfr= 0.0826(O.OZX1ZOO)rO.OZr2
- \ '\ - 't
f = 0.016 ' =10.442m
0.15s

,- _ 0.0165(3000) 1.912 Energy equation between A and B:


0.2 2(e.81)
. pB
!-A- + z^-hrt+ HA-ru= - + zB
h1= 46 m
* * T
0 + 0 + 10
- 1.033 + HA _ 10.442= 0 + 0 + 60
Powerrequired, p=eyHL HA= 6L.475m
= 0.06(e.81)(46)
Power required, p = 27.-1, Kilowatts
Power delivered by the pump (output power)
P=QyHA
= 0. 02(9,81 0) (61,.475)
=12,051, Watts x (1,hp/7a6 Watb)
P =t5.17Hp
A?lL CHAPTER SEVEN
.r t Fluid Flow in pipes FTUID FLUID MECHANICS CHAPTER SEVEN
& & HYDRAULICS Ftuid Flow in Pipes 435
Pressure at O:
Energy equation betweenA and O:
Solution
Ea-hfi=Et
__2

zgy''29 +zo-rrn=r'r' *
+-*PA 4Pt
v
t7.

o+o+10-1.033- 8(0'02)2 + Pr +0
n2 g(0.2)a v
pr = 87.76kPa

Pressure at €): Q = 0.15 m3/s


C=120
Energy equation between @ and B:
Ez-hp=Ee
-.2 Frictional head lost, /y=
+.**r,-t,P=o!-*lt
-E't2gy'6 *ro
1 0'67(] (0'1 5)
Frictional head. lost, /,/ = 9-0) :-8s
8(0.02\2* D^ (120)1'85 (0.?s)4'87
* o -70'442= o + 6 + 6s
;T(r"f ; Frictional head lost, U= 3.89 m
pz= 690.4kPa
Energy equation between 0 and G) (Datum at El. 0)
Fr-HL-HE=Ez
0 + 0 + 197 - 3.89 - HE = 0 + 0 + 50
- 41 (CE tuovemEEF HE =143.11m

Power, P=QyHE
= 0.15(e.81)(143.11)
Power, P = 210.59 kW (lnput power)

Power generated (output power)


P=210.59x85%
P =179kW

Problem 7 - 42 (CE November 2002)


The pump shown in the Figure draws water from a reservoir and discharges it
into a nozzle at D. The length of pipe from the reservoir to the pump is 150 m
and from the pump to the nozzle is 1500 m. The pipe diameters b_efore and
after the pump are 450 mm and 600 mm, respectively.

The atrnospheric pressure is 95 kPa absolute and the vapor pressure is 3.5 kpa,
Use / = 0.02 for hoth pipes. zr = 4 rn. The pump is to operate such that the
discharge will be the maximum possible.
CHAP1ER SEVEN
436 Fluid Flow in pipes FLUID M
FLUID MECHANICS CHAPTER SEVEN

Determine the maximurn


&
& HYDRAULICS Fluid Flow in Pipes +37
rate at which water may
reservoir? be pumped from a

7.667 Y- = 5.327 m
D 2g
a = 3.692rn/s

Maximum discharge, Q= Aa
= + (0.4s)z (3.692)
Maximum discharge, Q = 0.59 m3/s

Assume that 5T liters per second of oil (p = 860 kg/m3) is pumped through a
300 mm diameter pipeline of cast iron. If each pump prodrr.ur 6g5 kpa, how
Solution far apart can they be placed? (Assume/= 0.031)

Solution
Each pump must be spaced such that the head lost between any two
pumps is equal to the pressure head produced by each.

L 685 x 103
Pressure head., = = 81.2 m
v 850(9.81)

,. _ 0.0826fLQ2
"f- -_-;,s
0. 0825(0. 031 ) L(O.Os7 )2
81.,2=
since the pump is above, the
water surface of the source L=23,718m=23.718km
at the in_let (at B) is always tank, the pressure
,r"gr;;;
1i,;;il).
As the discharse incrs4s^s5,
the pressure at B drops.
the absoture pr"urrrr. at B To avoid cavitation,
3.5 kPa.
musi;;; i"x il;; the given vapor pressure
of For a 300 mm diameter concrete pipe 3,500 m long, find the diameter of a 300-
m long equivalent pipe. Assume the friction fictor f be the bame for both
lnergy equation between A and B: pipes.
(using absolute pressure
ana darim at A;
Ea-hras= Ee Solution
uo2 Po ,, For an equhtalent pipe system, the head loss anil
7),2 n-
29+ y *-'^ fL
D 2g= 4 *i*ru oiginal ptpe systgm.
florurate must be the same as the

o+ jr .o oX?1150)
)
* = ** # *o Qo= Q,
hp=li )
Eq. (1)
Eq. (2)
.HAPTER s.:vP;lres
439 FLUID MECHANICS CHAPTER SEVEN
& HYDRAULICS Fluid Flow in Pipes 439
0.usfirfJ"# _ o.e$erJ,* Solution
. Dou Drt Using Manning's Formula for circular pipes;
Lo
Do'=L, ,-,- L0.29n2 Lp2
D,u tU- D16/3
3,600 300
3005 D,u -"_lrfL -t0.29n2e2
D"t6/3
D. = 182.5 mm
For pipes in series, Qt = Qz= Q

7- For pipe 1:
Two pipes, each 300 m long, are connected
in series. The flow of w n_ 10.29nr2Qr2
through the pipes is 150 lit/sec with a total n 76/3
frictio.al loss of 15 m. If one r -t---
u7
has a diameter of 300 mm, what is the
diameter of the J;;;p;
minor losses and assume/= 0.02 for both pipes. 1.0.29nr2Q2
-
(o'sltolr
Solution
$= 41.4.87 n* Q2 ) Eq. (1)

- 1 pipe pipe 2
l00m_300mm 300m_D=? For pipe 2:
n _ 1}.29nr2Qr2
oz-
Qr
- -
Qz
4Ut
Q, = Qr= 0.15 m3/s _ 70.29(2nr)2 Q2
HL=h7+h12
D116/t

"_
41..1.6nr2Q2
_ 0.0826(0.02)(300)(0.15)2 0.0826(0.02X300x0. 15) 2 oz- ) Eq. (2)
1s
0.3s
+ -rjut
Ds
D = 0.255 m
D=255mm lSr = SzI
a+ry
41.L6
414.87 tt&ff= --Dvr,-
-46 Dz = 0.548 m
Two pipes 1 and 2 are in series. If the roughness coefficien ts n2 =2rlr and D2 = 648 mm

fffi:;,,lt;,i:r,ffi rind the diameter 6, tih;;;;;il",i the


*".gy s,"a"
Problem 7 - 47
Two pipes L and 2 having the same length and diameter are in parallel. If the
flow in pipe 1 is 750ht/sec, what is the flow in pipe 2 if the friction factor/of
the second pipe is twice that of the first pipe?
44o Ffif,'J:l',:""5}", FLUID MECHAN'CS CHAPTER SEVEN
& HYDRAULICS & HYDRAULICS rr"ii il.*i"],,p"r +41
Solution
For pipes ir-r parallel, the head losses are equal 0.0826(0.02) (3000) (0.01)2
lrfl = = 1.549 m
tfi = hfz o.2s
o.wf1w12 _ o.o8iw2L2Qr2
or'/ ln= 0.0826 (0 .02) (2200) (Q )2
Y(}fi) Dr5-/
0.35
= 1495.64 Qz2
f, Qr'= Qz2
(0.ZS1z = 2 Qzz 0.0826(0.02)(3200)(Q3 )?
lr\ = = 16,520 Qtz
Qz = 0.53 m3/s o.2s
Qz = 530lit/sec
0.0s26(0.02)(2s00)(0.01)2
lrfn = = 0.0452 m
0.4s

A. pipe network consists of piperine 1 from


A to B, then at B it is connected to lhn= hrl
pipeli^es 2 and 3, whereit merges again at 1,495.64 Q22 = 16,520 Q32
Joint C ,;;;;;r"gi" pfp"lr-r""I Qz=3.323 gt ) Eq. (1)
up to point D' pipelines 'r',2 and4 are in ,"ri",
.o.rr".fio" *i;r;;, pipetine, z
and 3 are paraller to each other. If the rate
of flow from A to B is id ut"rr7r"" lQz+ Qs = 0.011
and-assuming f = 0.oz for a, pipes, Determine
the flow in;";h pip" and the 3.323 Q3 + Qs = 0.01
total head lost from A to D.
e3 = 0.00231. m3/s
Qz= 2.3't'us

Substitute Qr to Eq. (1):


Qz= 3.323(0.00231)
= g.QQ/g$/ yyP/5
Qz=7.687 as
IHL = hfl + hp+ lql
HL = L.549 + 1495.64(0.0076842 + 0.0452
Solution HL = 1.583 m
Qz
.---->
Problem 7 - 49
A pipe system, connecting two reservoirs whose difference in water surface
elevation is L3 m, consists of 320 rn of 600 mm diameter pipe (pipe 1),
branching into 640 m of 300 mm diameter pipe (pipe 2) and 540 m of 450 mm
diarneter pipe (pipe 3) in parallel, which join again to a single 500 mm
diameter line 1300 m long (pipe 3), Assuming/= 0.032 for all pipes, d.etermine
Qt=Qq-1,0L/s the flow rate in each pipe.
Q, = Qo = 0'01 m3/s

o'0s26{9'
1tr1- 1
442 F,Xf,'J:1i:,iI", FTUID MECHANICS
& HYDRAULICS ,,:iH,lJ,:?Y# 443
Solution
104.32Qr2 = 13

Qr = 0.353 m3A
Qz= 0.266(0.353) = 9.994 m,
Qt= 2.756(0.094) - 0.?59 ms/s
Qt= Qt = 0.353 m7s

Pipe 3
640 m Problem 7 - 50 (CE May 2002)
450 mm
For the pipe system shown in the Figure, n = 0.015 for all pipes and the flow in
pipe 4 is L2 cfs. Determine the following:
Qt= Qt ) Eq. (1) (a) the head lost in pipe 1 in feet,
Qr=Q2+P, ) Eq. (2) (b) the total head lost in terms of the total discharge e, where e is in cfs.
lrfr = llf, ) Eq. (3) (c) total head lost in feet.
HL = ltfi + fufz+ ttfq = 13 m ) Eq. (a)
@

_ 0.0826l/LQz
t,f
'ps
1500ft-24in.
t,fr= =1o.Blze12

Itfi =
--S48L
0.0826 (0.032) (6 40\Q,2
5000ft-12in.
= 696.15Q,2
(0.3)s
0.0826 (0. 032) (6 40 \ Q a2
hfi= = 91,.67Q"2
(0.4s)s Solution
0.0826(0.032x1300)Q4 2
lrf. = = 44.19 9oz
(0.6) -,-f. Qz

In Eq. (3): -
696.15Q22 = 91..67pr2 1500ft-24in. 2000ft-24in.
-
Qt= 2.756Q, o- ---_+ D
Qr Q,
In Eq. (2): 5000ft-12in.
Q, = Qr+ 2.756Q2
Qt = 3.756Q2 ; Qz = 0.266Q1

In Eq. (a) Q= h= Qa=12fF/s


HL = hfr + hfz+ hf+= 13 e = A t, = A1'49 Rz/i st/z English Version
n
10.877 Qf + 696.1,s (0.266er)z + M.19 (e1)z = 1g
aa,ll
r r r
CHAPTER SEVEN
Fluid Flow in pipes FLUID MECHANICS
FLUID MECHANICS CHAPTER SEVEN
& HYDRAULICS & HYDRAULICS Fluid Flow in Pipes 445
a= o, 7Aj (D / +yzrt (HL/ L)t/z Problem 7 - 51
;
4.632n2 Lo2 water is flowing at the rate of 300 lit/sec from A to E as shown in the figure.
rrl = -----_-___--___:_ in feet Compute the flow in each pipe in lit/sec and the total head loss. Assurie/ -
/3
D16
0.025 for all pipes.
(r) Head lost in pipe 1:
4.637 (O.C[:qz QsoO)(12)2
HL1_
(24/12)16/3
HLr = 5.59 feet

(b) Total head lost in terms of


e:
Total Head Lost, HL= HLr i HL2+ Hl-a
Total Flow = Qt= Qq= Q
4,637 (0.01'q2 $soqQz
HLl_
(24 / tz71e /t Solution
HL1= [.Q]$$ Qz
4:,637 (0.07q2 eooo)Q2
HLA_ 1500m-300mm
(24 / 12)16/3
HLa = Q.Q$l$ Qz
B Qr-. g Qr-.
IHLz= HLsl
4.637 (9_.9-1?F g.ggqar, _ 4.037(0,01s)2 (s000)e32

^ (18/rz7tt/s @
Qt = 0.3034 Q,

[02+Qs=Q] Qr=Qe=0.3 e Eq. (1)


Qz+ 0.3034 Qz= e Qr= Qz+ Qs+ Qt + Eq. (2)
Qz= 0.7672Q Qs+ Qt= Qs ) Eq. (3)
hfz= hfi+ hfs ) Eq. (a)

HLz _ 4.637 (0.01q2 $ooo) (0.7 672Q)2


hfi= hfq ) Eq. (s)
HL = hfi + hf2+ hft ) Eq. (6)
HL2= 0./$lg gz
,_,_ 0.0g26fLQz
ftl--
y3 = g gqls Q2+ 0.ls26 02+ o.os18 e2 "D5
HL= 0.3732e2
0.0826(0.02s)(300)(0.3)2
G) Total head lost: h/
- - = 3.02 m
Total head lost = 0.3232(1,2)z = S3.Z4feet (0.45)"
0 0825(0':0?s]!15q0)Qr'?
h, = = 1275e22

hfs
_ 0.0826(0;025X600)Qa2
=1269Q*
(0.2s)s
CHAPTERSEVEN CHAFTER SEVEN
^ ^,
++O FTUID MECHANICS] FLUID MECHANICS
Ftuid Ftow in pipes 6. HYDRAULICS & }IYDRAULTCS Fluid Flow in Pipes 447
0.0826(0.025X600)Q42 Pipe Data
lf4= =3871Qq2
(0.2)s
Pipe tensth; t (m) Diameter, D (mrn)
0.0s26(0.02s) (600)Qs2 7 450 600
hfs= = 51_0Q52
(0.3)s 2 600 500
2 J 360 450
ry6
_ 0.0826(0.025)(400) (0.3)
= 4.03 m 4 480 450
(0.45)5 500
5 540
In Eq. (5)
1269Qrz = 3871'Qq2
Solution
Qt= 1.747Qq
Qr Qs
In Eq. (3) A sL

7.747Q4+Q+=Qs
Qs= 2.747Qa
In Eq. (a)
1275Q22 = 1269 (1.747Q4)2 + 510 (2.247Q4)2
Qz= 2.4612n
In Eq. (2)
Qt = 2.467Qq + 7.747Q;+ Q+ = 0.3 Qr=Qs ) Eq. (1)
. Qt=0.0576ms/s Qr= Qz+ Qt ) Eq. (2)
Qz= 2.46'1.(.0576) = 0.7418mt/s Qs= Q,+ > Eq.(3)
Qt= 1.747(0.0576) = 0.1005 m/s hp= hp+ hya ) Eq. (a)
- Qs= 2.747(0.0576) = 0.1582 m36 HLar = hn + lrp+ hO= 15 ) Eq. (5)
Check:
Qt= Qz+ Qr + Qr .
llr=-
o.o826fLd
0.3 = 0.1418 + 0.1005 + O.OSZG = 0.3 (OK) 'D'
iil 0.0825(0.02x450)Q12
Ur= =e56Q?
(0.5)s
1 Problem 7 - 52 (CE Board)
0.0826(o.02x6oo)Q22
The total head lost from A to E in the figure shown is L5 m. Find the d.ischarge h,= = 3].72Q22
in each pipe. Assume f = 0.02for all pipes. (0.5)s
0.0825(0.02)(35qQ32
hp=
-- (olq'- =32.23Q*

0.0825(0.02x480)Q42
hn= '(0.4s)5 = 42.97Q*
)
5
= 1'1..47Qs2
(0.5)s
A/LA
I r rL, CHAPTER SEyEN
FLUID
Fluid Flow in pipes
FLUID MECHANICS
& CHAPTER SEVEN
& HYDRAULICS
In Eq. (5): Fluid Flow in pipes 449
15 = 9.56Qrz + 3'.1.32e22 + l.t .47es2 Solution
But Qs = el, Fronr Eq. (1) Note: The additionar pipe should be
laid in pararer (not in series) with
the original pipe in ordlr to i""ruuru
15 = 21.03Qr2 + 31.72e22 ) Eq. (6)
tf,u.*"pr.ioy of the system.

ln Eq. (4):
37.72Q22 = ZZ.2ge32 + 42.97 ei .lAddiuonat pipe o
///lt, = (2/3)f
But Q, = p,
37.72Q22 = 75.2Q32
Qt= 0.649Q2 ) Eq.(D -
In Eq. (2):
Qr=Qr+0.649Q2 Original pipe: piue O
Qt = 1.649P, ) Eq. (8)
Capacity, Q I e,
o'o826fLQ2
In Eq. (6) Head lost, H -
15 = 21.03(1. 649er7z D5
+ 31.72e22
15 = 88.eQr: Head los1, g =
Qz= 0.411 m3/s Dru
Qt = 0.649(0.41,t1 = g.2U, m3/s = o,
er = 0.679 m3/s = pu <{ , Additional pipe: pioe @

Check:
[::tr],:r'iry' Q'= 1 5Q'
- Q, = 0.0826U? / 3) f
ez + et = 0.41.7 + 0.267= 0.62g (OK) H= )L(1.sQ 1 )2
D2

[H = ]4 (since they are laid in parallel)


A.n existing pipeline 0.08?6. JLe12 _ 0.0826l(2 / 3)
is to be reinforcj witl f)L(l.seiz
I
pipe rrictioir zze ,] *r.'" ord
o^e. o r*'-,i'";il ;n;il"j;"""r'flTHI Dr'
of the ord one and trre additionrir;;;f
"" D.s -D2
capaciry i, 16o?"-;;;"
brs -
Z F

ffff:t#",Tr',:fnj|m- pipu
'no.,i'Juilo*pu,ua to ure ori
exisring
one. use the
-n
'''
D"
= 1.08
=4
u7
Therefore, Dz = 1.0g times Dr

with velocity of 1 m/s in the 20,-mm-diameter


calculate the flow through the system
pipe in the figure rhownp
,.,a ,t-' nuua il ,"q,rir.a,-'it,uma r
0.02 for all pipes and neEflect
rnirro, torrur.-- " /
4so Fllf,'J:li'"",'#, FTUID MECHANICS CHAPTER SEVEN
& HYDRAULICS Fluid Ftow in Pipes 451
[Qr = Ar ur]
Qr= i (0.2)'?(1) = 0.0314 m7s

,_ _ 0.0826(0.02x300x0.
,ti,-T=I.cJn 031.4\2

From Eq. (3):


h12= hn = 1.53
0.0826(0.02)_(3OqQ22
hD_
' = 1.53 m
0.3c
9r= Q.Q$$$1p11s/s

Pipe Data
From Eq. (1):
Pipe Leneth, L (m) Diameter; D (mm) Qr+ Qz= Qt
1 300 200' Qc=0.03L4+0.0856
2 300 300 Q3 = 0.118 m3/s
3 300 500
4 600 300 From Eq. (4):
5 800 300 hfi= hF
0.0825(0.02x600)042 _ 0.0826(0.02)(800)Qs2
Solution
o.3s o.3s
Qs=0.865Qt ) Eq. (6)

From Eq. (2):


Qt= Qq+ Qs
0.118= Qa+0.866Q+
Qr=0.0632nt/s

From Eq. (2)


Qs=0.118-0.0632
Qs = 0.0548 m7s
Givenar=1m/s
From Eq. (5)
Equations:
H = hn +.hfi + hIa
Qt + Qz= Qt ) Eq. (1)
0.0825(0.02x300x0.1 18) 2 0.0825(0.02x600x0.06,32) 2
Qt= Qa+ Pt ) Eq. (2) = L.53 +
It1 = hp ) Eq. (3) 0.5s 0.35
hfl= hfr ) Eq. (a) H=3.38m
HL=lt1+lryt+lV+=H ) Eq. (s)
+s2 F,:fioJ:[1'""o,5]",
FLUID MECHANICS
& HYDRAULICS ,,:lHIff:?Y,T 4s3
Problem 7 - 55 (CE November 1983)
,_ _ ___;_T€Ti_
r,rE_
10.29n2LrQrz
Three pipes of different lengths and diameters connected in series as
Up
discharges 160 liters per second. If the roughness coefficient n = 0.012
disregarding miiror losses, determine: 06 1 t_ 10.29(0.0142(4,100)(0.16)2
(a) the head loss in each pipe, :
(b) the diameter of an equivalent single pipe that could replace all Dr=0.304m=304mm
three pipes, and
(c) draw the approximate EGL and HGL. (c) EGLandHGL:

AoL-qcoD
450 mm - 1800.m 300 mm - 1500 m 250 mm - 800 m hn ='1.83

vt2l29 ha = 34'98
Solution
Qr=Qr=Qt=0.16rrt3/s v22/2g\ \ \

(a) Head loss in each pipe:


,. _
.1
1.0.2%iLQ2
l,r=- (Manning's Formula)
DLG/3
2
L0.29(0.0142 (1 S00 Xo. 1 5)
hn=
og
h7 ,150 mm
- 1800 m 300 mm - 1500 m 250 mm - 800 m

10.2e(0.012)2 (1500x0.16)2
It" =
/3

hp= 34.98 m
Q3)16
illlil#;l:,i'#T"[:T,l..?:il designed ror riring tank hucks with
water. The 10-inch line has an over-all length of L00 feet. The 5-inch line A is
10 feet long. The 10-inch line B is 40 feet long. The Darcy-Weisbach factor/,
,. _ 70.29(0.012)2 (800X0.15)2
,,tr-@ equals 0.02. Neglect minor losses. Determine the total digcharge which can be
delivered by this systemwhen all the gate valves are fully open.
hp= 49.33m

(b\ Equivalent pipe:


p. = Q.lg pr/s
It6=HL=lyt+|ry+hF
h6= 4.93 +34.98 + 49.33
hp= 89.14m
Le = 1,800 + 1,500 + 800 = 4,100 m
CHAPTER SEVEN
454 Fluid Flow in pipes FLIJID MECHANICS CHAPTER SEVEN
& HYDRAULICS Fluid Flow in pipes 455
ut
tL2-A _
t -
0.02(10)
(6 /12)
HL24 = 0.761 QA2

29 s2.2(6 /744
o^2
= 0'403 9^z
z8
In Eq. (1);
80 - 0.125 Q2 - 0,161, Qa2 = 0.403
Qe2 + 20
0:125 pz + 0.564 Qa2 = 60 ) Eq. (3)
Energy equation between O and B:
, Er - HLt-z- HL2-s = 2,
,
o+o+80-HL1-2 -HLz-s= "j- +o+20
2g
) Eq. (a)

o,o2(40) 0.8'Lo6QB2
HLz-s=
(6/12) 32.2(6/n)4
HL2-B = 0.644 QB2

_ o.B7o6er2
au2
29 32.2(6 /n)4
,)

"!- = g.4gg 2rz


zg
Q=Qa+Qs )Eq.(1)
In Eq. (4):
Energ/ e_q_uation between 1 and A: q0 0,J25 Q2 - O.6u Qs2 = 0.403 Qs2 + 20
h- HLr-z- HL2-a= Bo
0.1X gz +'1..047 Qe2 = 60 ) Eq: (s)

0+0+ 80-HLt-z-HLz-,q= Subkact Eq. (3) - Eq. (s)


+O+20 )
+ Eq. (2) O.1'N 9z + 0.564 Qaz = 60
- 0,125 Qz + 1.012 Qrz = O0

29= sDn:iluoun'
r.rot",4 8Q2
nz-i;r_ 0.55e poz - L.047 Qsz = 0

Qe=1.352Q, ) Eq. (5)


fL a2 o.o2(1oo) o.81o6ez
D 29- OWn)
H11.2= From Eq. (1):
Q= Qe+ Qs
HL"t-z= 0.125 Q2 ".r@fu =1..362Qs+ Qa
Q= 2.362Qs
Qa = 0.423 2
f! F,Xf,'J:[T,;[,
From Eq. (5):
FLUID
&
MECHANICS
I,IVDRAULICS
CHAPTER SEVEN
Fruid Flowin Pipes 457
0;1%-91: 0.0826(0.02)(600)Q12
],047(0.42s Q)2 = 60 _ 0.0s26(0.02sx7s0)Q22
Q = 13.E5 ft3ls
0.15s o.2s
Qz= 7.6422Qy

Qt + 1.6422Qr='1.
(f = 0.01s), 150 m tong, is connected
fl,T;ru;*;:j#ry" etel pipe- (f = 0.02),200 m llong.
in series
Out".o.in"1n ai,
Qr=0.3785nf/s
o6 an a^,,i.,_r^..r
or -:_ -, pid"'";
equivalent singre
the two pipes.
t"r" ffi; ffiit: ,:rT'ilT""ffj HLo = tq-
0'0826(0'02X60-0X0'3785)2
= 1870 m
0.15s
Solution For the equivalent pipe:
Setp=1m3/s qr= 1rr,P/s
HLr= HL" = 1870 m
For the original_pipe sys-t:m 0.0826(0.0]s)(7s0)(1)2
(two pipes
eo=er=ez=1mr/s ' r r----in series): HLE_
DE
= :ilJ70
HLo=ltfi+hp
Dt= 0.278 m = 218 mm
_ 0.0826(0.015X1s
0.0825(0.!2x2oox1)2
+
HLo=1,222.81m
"*-U ln the figure shown below, it is desired to pump g,Al't,OOO lit/ d,ay of water
For the equivalent pipe:
from a stueam to a pool. If the combined pump and motor efficiency is z0%,
Qe = 1m3/s calculate the following:
HLe=7,222.8'1. m (a) total pumping head in meters,
91, = 0.0825(o'o?(3so)ff (b) the power required by the pump, and
=.1,222.87
. (.) the monthly power cost if electricity rate is p5.00 per kw-hr. Assume
De=0.226m=225mm that the pump aperates for 24 hours and take 1 month = 30 days.

Length Diameter Hazen


Pipe
(m) (mm) G
A l5O-mm-diameter 1 1.,525 200 100
"p"_(=
diameter pipe 6= 0.d25),750 -d#rri-",*,e
O.O2),600 m long, is in parallel with a 200_mm- 2 1,525
single pipe of length 750 m
* b;;. diameter of an equivarent J 915
200 110
and/= 150 740
fiOfS ,i.r"i""uld replace the two pipes.
Solution
SetQ=1m3/s

For the given pipes (two pipes


in parallel):
Qr+ Qz=1
h1= 14
4sq F,:f,'J,:[::,:]", FTUID MECHANICS

Solution
& HYDRAULICS .,,;;;;;i;;il;
CHAPTERSEVEN
+se
HLAB=14+lxo ) Eq. (t) Power required by the pump (Input power):
Qt = Q2+ 2, ) Eq. (2) P"= Qy HA
hfr= hfi ) Eq. (3)
= 0.03948(9.81X69.8)
Hazen Williams Formula: P,= 27.03 Kilowatts
7o'67LQ1'ss
h,=
'
Power input = P, / Efficiency
C17'8sD4'87 =27.03 / 0.70
Power input = g8.6l,4kilowatts

Power cost:
Cost = Power input, kW x Time in hours x Power rate per kW-hr
= 38.614 kW x (30 x 24hr) x (P6.00 / kW-hr)
Cost = P1.66,872.48
Qt = 3,471,,000 lit/ d,av x^(1, day 24hrs)
Qt = 39.4gL/s = 0.0i9+S mr/s
/ x' (1 hrl3600 sec)
\r r'r
Solving for e2 and Or: Problem 7 - 60
From Eq. (3): How many liters per second of water must the pump shown suppry when the
lhn= hn] flow needed in the 91S-mm-diameter pipe is ,.rrn 1 s? Assume / = 0.017 for
70.62(1,52qe21.8s all pipes. ^z
_ rc.67(91s)p"r.as
(110)r'u516.2;r.az
fr+orTtr(015J?:7
Qt= 0.7879,
From Eq. (2):
[Qr=Qz+Q.,]
0.03948 =ez*0.2g7e2
Qz = 0.0221 m3 / s

From Eq. (1):


70.67 (7,525)(0.03148; t'as
HLAB = 70. 67 (7,5251 (0. 0221 )
------------------1-
1.8s

(110) I'u519.2;r'az
HLAB = 26.807 m

EnSrgy-gquation betweenA
and B:
Ea-HLaa+HA=Ea
. *,^'-rr^,+ HA= . PB +
* ? { Y
zB
0.+.0 + 47 -26,.g01 + HA= O. o.'n!O
HA = 69.gm ) Totalpu*pir,g;ra
+60 F,Xil?,:[i:",:]", FLUID MECHANICS
& HYDRAULICS
CHAPTER SEVEN
r,'iiJi,l"i,"i,'rp"' 461
Solution
o.osze e.otz ) (tszg)e
hrn = ^2 = 9.28
o.5ogs
Qa = 0.35 m3/s

At junction B:
Inflow = Outflow
Qr+ Qt= Qz+ Qt
Q1 + 0.35 = 1,.31,4 + 0.1.42
Qq = 1.106 m3/s
\ --- Qt = 1,106 Liters per second
1 El. 12.2 r

Problem 7 - GL
The turbine shown is located in the 350 mm-diameter line. If the turbine
efficiency is 90%, determine its output Power in kilowatts.

Llne 1:
1220m-350mm
Q2= 1.374m3/s f = 0.018

_ 0.0826(0.077)(24!0)(1.314)2
'
hp
0.9L5'
= e.22 m

Elevation B' = 6.1 + luz


Elevation B' = 6.'L + 9.22 = 15.32 m

16 = EIev. B' - EIev. C = 15.32 - 72.2


hp= 3'72
0.0826(0.077\(122qQ32 _
ha=
' 3.12
0.4055
Qr = 0.742m3/s

hp=Elev.D-Elev.B'
hn= 24.6 -15.32= 9.28
CHAPTER SEVEN
462 Fluid Flow in pipes FLUID MECHANlCS CI.IAPTER SEVEN

Solution
& HYDRAULICS & HYDRAULICS I iurd Flow in Pipes 463
Porver input = Qty HE
hfr + HE = 0.2787(e.8 I )(5.-lt5)
I Power input = 1.1.83 kilowatts
J Dl
--
|
hi, -./ Power outl)ut = p1,u,cr. input x Efficir'rrcy
II rt sto
zzn-m /
// - 14.83 x 0.90
rt.
J fl^^.r.^...n Power output = 13.347 kilowatts
Llne 1!
1220m-350mm
f = 0.018

Llne 2: A 1.,200-mm-dian'reter concretc pipe 1,800 n1 long carries 1.35 m3/s from
610m-150mm
f = 0.024 reservoir A, whose water surface is at elevatitrn 50 rn, ancl clischarges into two
concrete pipes, each 1,350 m long and 750 rnm in diameter. orre of the 750-mm-
Line 3:
diameter pipe discharges into reservoir B in whiclr tlrc water suri.rcc' is at
Qz= 0.23 m3/s 2.140m-3O0mm elevation 44 m. Determine the elevation of the water suri.rce of reservoir. C into
f = 0.02
ha _ 0.0826(0.02) (2440) (0.n)2
Qr = 0.23 m3/s whichtheotherT50-n"rm-diarneterpipeisflowing. Assunre/=0.02forall pipes.
' 0.3u-
hp = 87.75 m Solution
EIev. D' = EIev. C + hn E|. 50 m
= 2g0 + g7.75
EIev. D' = 367.75 m

hp = Elev . D' - EIev. B 367 .75 _


3ZO
lry=37.75m
o'0825(0.9?!)(610)Qz2
1,, = = 37.75
0.155
Qz= 0.0482 m3/s -q
At junction D:
[Inflow = Outflow] Qr = 1,35 m3/s

Qr= Q2+ g,
= 0.0487 + 0.29 \ El.?
2
Qt = 0.2787 mt/s 0.0825(0.02x1s00x1.3s)
hp
1..2s
hn=2.18m
hn=
=26.8:25m
Elev. P' = Elev. A - hn= 50 - 2.18
hn + HE = Elev. A -Elev. D, Elev. P' = 47.82m
26.825+HE=4OO-367.75
HE = 5.425 m h1z = Elev.P' - Elev. B = 47.82 -M
hp=3.82m
464 CHAPTER SEVEN
Fluid Flow in pipes
FLUID MECHANICS CHAPTER SEVEN
& HYDRAULICS Fluid Flow in Pipes 465
4p=ry=3.82 h7 = 62-4 m
Qz= 0.6875 m3/s 0.0826(0.02)(150qQ12 _ 62.4
At junction p: 0.6s
Qr = 1.399 m3/s
[lnflow = Outflowl
Qt = Q2+ 9, 2
1.35=0.5325+e3 o.riszo(o.ozs) (rooo)e,
lq> = l$.6 =
Qt = 0.7125 mt / s (0.4s)s
Q2 = 0.488 m3ls
0'0826(0.02)(1,350)(O.ztZqz
t*=
' .,:^
lrsS----.--- = 4.77 n Qt= Q2+ 9,
Qt = 1.399 - 0.488 = 0.911 m3/s
Elev.C=Elev. p,-h,
= 47.g2 _ 417
EIev.C=43.05m Eleuation of reseruoir c:

lrft = = 35.54 m
El. C= 870.6-35.54
A, B, and C are connected El. C=835.06m
I[."_"^.::"*oirs r

l',:Hi:'ff ffi :*L*:i;:::ff "? Problem 7 - 64


"";S{Ij:5:U_X",i"T.,";l,li,3;f
ffi : ,,ei{;::ffi
6; tilr
*; #; r,i: fr,iiu;r,"'*:;::*;i
;,*;;* ;#;tT:; fl7',"'5,",;;1,"ffi
Determine the flow in each pipe in the figure shown and the elevation of
reservoir C if the inflow to reservoir A is 515 Lit/sec.
k?11#,
Solution
El. 933 m

300m - 450 mm ,

f = 0.03

P: El 366m
600m - 600 mm
P = 4,950 kPa
f = 0.025
CHAPTER SEVEN
466 Fluid Flow in pipes FLUID MECHANICS CHAPTER SEVEN
& HYDRAULICS
467
Solution ',iiJLi-ili,ipii
Elev. P = Elev. E' + hfi= 84.?3 + 7.37
Elev. F' = 91.6 nt
/rp = Elev. F' - Elev. D = 9'1.6 - 90
hp=1'6^

k F
El.

Qs

300m - 450 mm
f = 0.03
9Or;

O
hfr
,,6 _
0.0826(0.03)(300)Q12

Qs

At junction F:
[Inflow = Outflow]
Qr= Qt+ Qs
0.45'
= 0.199 m3h )
_ 1.6

Flow in pipe 3

= 0.247 + 0.199
Qr = 0.445m3^ ) Flow in pipe 4
El. 80 m
"%f
6"""'
0.0825(0.03)(390)(0.446)2
,,,, - = 8.01 m
600m - 600 mm
'' 0.45'
f = 0,025
Qr = 0.515 m3/s Elev. C = Elev. F' + hfi= 91.6 + 8.01
Elev. C =99.6'l.m
lq=
Problem 7 - 65
Elev. E'= EIev. A + hn = g0 + Determine the flow in each pipe in the three reservoirs shown.
4.2g
Elev. E' = 84.23 m

/rp = EIev. B - EIev. E, = 90 - g4.23


\z= 5.77 m
_ 0.0826(0.03) (600)Q ?2
'.!z
hD
-----[fii* =s.n
Qz= 0.268 m3/s ) Flow in pipe 2

At junction E:
Il"flo* = Outflowl
Qt+ Qz= Qt
Qs=0.515 -0.268
Qt= 0.247 m3/s ) FIow in pipe 3

0'0826(0.03X900X0.247\2
1J=
;,o ___
=7.37 m
0rr5__
q6s F,Tf,'J:l::1f , FLUID MECHANICS _!f'l[rER.SE^YEI
Fluid Flow in PiPes 469
& HYDRAULICS
Solution

lrr =

.lt^=
0.0s26(0.02) (1800)Q1
o.4s
0.0826(O.OZ5)(ZOOOIO"2
2
=2e0.4Qf
T
\ ,\ ,-z I
=1D^nn_l
Lr''L Y:2
0.55 hn=x
0'0825(0'92(4009)Q22
ftr - = 30.2s e3z o\ P'

^x
Direction of flow:
The flow in each pipe is due to gravity. The flow in pipe 1 is obviousry hr: = 70 'x
away from reservoir A and the flow in pipe 3 is towardi reservoir
tl're flow in pipe 2 is either away or towaids reservoir B.
c but
To d.etermine the
direction of Q2, assume Q, = 0, then hp = 0 and the EGL for pipe 2 is
,,,2 //Q,.

horizontal.

x;
l-[
=30m
hn
hp
=
=
29g.q Q? =
30) Qz = A.087 ^l; -fr
132.2 Qzz = x -
Qr = 0.0587 \E

hp=30.25 Qsz=70 - x; b= 0.152^mA


At junction P:
[Inflow = Outflow]
Qz=o I Qr + Qz= Qs
hn=40m o.oss7 J;
+ 0.087 JT--so = 0.1s2
^m
4
/Qr
,, ,H
.square
JV * t.Esz,Fgo = 3lJVo-x both sides

x + 2.964 J; JrSO + 2.196(x- 30) = 9'61'(70 - x)

2.g64J; J, - 30 = 73s'5s - 12'806x' square both sides


8.785(x)(r - 30) = 545,500 - 18,917x + 164 xz
lSS.ZtS* - 18653r + 545,500 = 0
htr = 30;
290.E Q,f = Qr = 0.321mt/ g x=50.287 m
hr = 3O.N Q* = 40; Qs = 1.15 m3ls

Since Q3 > Qr, the supply from reservoir A is not enough for pipe 3.
Qr= 0.0587 J50.287 = 0.416 m3/s
Therefore, Q2 is away from reservoir B and p' is berow reser-ioir B. Qr= 0.087 JWn = 0.392 m3/s

er= g111Jfl-irf,}fF = 0.808 m3/s


CHAPTER SEVEN
470 Fluid Flow in pipes FLUID
FTUID MECHANICS CHAPTER SEVEN
& & HYDRAULICS Fluid Flow in Pipes 47t
Check:
Qr + Qz= Qs Direction of flow:
0.416+0,392=0.g08 Assume Qz = 0:
0.808 = 0.808 (OK)

7- hn=15m
flow in each of the pipes shown in the
fftfiHl':e figure. AssumeT= g.g2
_J
I

hm=25rn
,{\olEl
'o t\ Qr
ha=45m
ts*
o-
1
7

Q: El. 30 m

9O0m - 150mm

Ir7=1.2.75Qf =15; Qr = 1.085m3/s


lW=306Qsz =25; Qs= 0.28611.3/s
14=19,579 Qnz= 45; Q+ = 0.048 me/s

Qt + Qt = 0.334 m3/s

Solution Since Qr > (Qa + Qr), the


flow from pipe 1 is more than enough to supply
pipes 3 and 4. Therefore, Qz is towards reservoir B and p' is ,above
6' _
0,0825(0.02)_(5OUQr?
741/7' reservoir B.
0.65 -= 1)
LL.tr t1-2

0'0825(0'9.2)2!3oqQ22
hp - = LS4e e*
0.0826(0.02) (45qQ32
hF - = 305 Qs2
0.35
,- _ 0.0826(0.02x900)o, 2
- ------
,r,4
= 79,579 e4Z
0.1.55
472 Fi#!iI,l:"f,I",
FLUID MECHANICS
CHAPTER SEVEN
El. 90 m 6(HYDRAULICS Fluid Ftow," p;;"; 473
Problem 7 - 67
The pipe network shown in the figure
represents a spray rinse system. Find
the flow in each pipe. Assume Cl
=-120 foi at piper.

0.3 m3A
hr:=40-x
600m - 300mm 600m - 300mm 600m - 300mm
4 B c D
El. 50 m o E E
E E E
o
h
E
o
h O
E
oE
a\, ln r}
N N

al 1(:",,,t\ 600m - 300mm


o
o
E
!-
600m - 300mm
O
o
E
<f
E
o
O
rr-
a""" El. 30 m 600m - 300mm

0.1m3/s 0.1 m3/s


r Line 4: 0.1 m3/s 0.1 m3ls
9O0m - 15omm
lq1= 12.75 Solution
ef = x; Using Hazen-Williams formula;
lYz = 1,549 ez2 = 15
- g Qz= 0.025+JEi ,.
r=- 1.0.67L
hp=3o6Qt2=40-.,-; (r1'8sPa'az
Qt = 0.0S7zJ$-l
l\q ='1.9,579
e* = 60 _ x. Qt = 0.00715J60;
At junction p; =
9r= Q;+ et+ et
^o' 6ffiffi = 32'l' = Ksc= Kco= Ker = Krc = K,r
Qr-er_er_en=o Kau =
10.67(400)
= 520 = KBc = Kcr = Koe
{; _ 0.0254/G; _ oos7zJ6;
o.zs
_
(120)1'8s 19.2q+'tz
0.oo71s Je;:x =Q
Solve r by trial and error:
r = 3.055 m

0.28.,618 = o.4894
Qr = A
ms/s c
Q, = 0.025+J-rslfd-- Loop I Loop U Loop III
et = o.os72*=u#
Qr= 0.00715C0-B = 0.0$e;;
::::;:!r: 1,,
/\
(+ (+
lo,u
c
Check: 0,1 0.1 0.0s
F
0
t llf,,:^ : t:y;. 0 0s3 e
?K,.

También podría gustarte